Careers360 Logo
NCERT Exemplar Class 9 Maths Solutions Chapter 1 Number Systems

Access premium articles, webinars, resources to make the best decisions for career, course, exams, scholarships, study abroad and much more with

Plan, Prepare & Make the Best Career Choices

NCERT Exemplar Class 9 Maths Solutions Chapter 1 Number Systems

Edited By Ravindra Pindel | Updated on Aug 31, 2022 11:38 AM IST

NCERT Exemplar Class 9 Maths solutions chapter 1 provide detailed answers for questions related to Number System. These problems and their solutions are devised for a better knowledge of the chapter 1 of NCERT Class 9 Maths Book. These solutions are extremely useful in understanding the concepts and know-how of real numbers. Experts have created these NCERT exemplar solutions for Class 9 Maths chapter 1 with a thorough analysis.

The NCERT exemplar Class 9 Maths chapter 1 solutions provided for exemplar problems are detailed and expressive. These are helpful to pinpoint the critical understanding of concepts. The NCERT exemplar Class 9 Maths solutions chapter 1 are in accord with the CBSE syllabus for Class 9.

Also, read - NCERT Solutions for Class 9 Maths

NCERT Exemplar Class 9 Maths Solutions Chapter-1: Exercise-1.1

Question:1

Every rational number is:
(A)A natural number
(B)An integer
(C)A real number
(D) A whole number

Answer:

Answer: [C]
Solution.
Any number which can be represented in the form of p/q where q is not equal to zero is a rational number. So it is basically a fraction with non-zero denominator.
Examples: 4/5, 2/3, -6/7
(A) All the positive integers from 1 to infinity are natural numbers. These cannot be fractions.
Examples: 1, 2, 3, 4 and so on
(B) An integer is a number which can be written without fractional components or decimal representation. They can be positive, negative or zero.
Examples: \left \{ .......,-4,-3,-2,-1,0,1,2,3,4,......... \right \}
(C) All numbers that we generally use are real numbers. They can be positive, negative, decimal, whole, natural, integer etc.
Examples: 0.123, -2.5,0,2,5,6.3456
(D) All the positive integers from 0 to infinity are whole numbers. These cannot be fractions.
Examples: 0, 1, 2, 3, 4 and so on
From the above definitions we can easily see that every rational number is a real number. Therefore option (C) is correct.

Question:2

Between two rational numbers:

(A)There is no rational number
(B)there is exactly one rational number
(C)there are infinitely many rational numbers
(D)no irrational number

Answer:

Answer: [C]
Solution.
Firstly let us define a rational number.
Any number which can be represented in the form of p/q where q is not equal to zero is a rational number. So it is basically a fraction with non-zero denominator.
Examples: 4/5, 2/3, -6/7
So we can say that it is a type of real number.
Similarly we can see that irrational numbers are real numbers which cannot be represented as simple fractions.
Examples: \sqrt{2},\sqrt{3},\pi
So we can say that between two rational numbers, there are infinitely many rational numbers.
For Example: between rational numbers 5 and 4, there are rational numbers like 4.1, 4.11, 4.12, 4.13………and so on.
Therefore option (C) is correct.

Question:3

Decimal representation of a rational number cannot be :
(A)Terminating
(B)non-terminating
(C)non-terminating repeating
(D)non-terminating non-repeating

Answer:

Answer: [D]
Solution.
Any number which can be represented in the form of p/q where q is not equal to zero is a rational number. So it is basically a fraction with non-zero denominator.
Examples: 4/5, 2/3, -6/7
Terminating decimals have a finite number of digits after decimal point,
Examples: 1/2 = 0.5, 3/5 = 0.6
Non terminating decimals are the ones which keep on continuing after decimal point.
Examples: 1/3 = 0.3333..., 5/11 = 0.454545....
Recurring decimals are those non terminating decimals which have a particular pattern/sequence that keeps on repeating itself after the decimal point. They are also called repeating decimals.
Examples: 1/3 = 0.33333....., 4/11 = 0.363636....
Non-Recurring decimals are those non terminating decimals which do not have a particular pattern/sequence after the decimal point. They are also called non repeating decimals.
Examples:
\sqrt{2}=1.414213562373
\sqrt{3}=1.732050807568
\pi =3.14159265359
So by the above definitions we can see that the decimal representation of a rational number cannot be non-terminating non repeating because decimal expansion of rational number is either terminating or non-terminating recurring (repeating).
Therefore option (D) is correct.

Question:4

The product of any two irrational number is
(A)always an irrational number
(B)always a rational number
(C)always an integer
(D)sometimes rational, sometimes irrational

Answer:

Answer: [D]
Solution.
Irrational numbers are real numbers which cannot be represented as simple fractions.
Examples: \sqrt{2},\sqrt{3},\pi
The product of two irrational numbers can be rational or irrational depending on the two numbers.
For example, \sqrt{2}\times \sqrt{2} is 2 which is a rational number
whereas \sqrt{2}\times \sqrt{3} is \sqrt{6} which is an irrational number.
Therefore option (D) is correct.

Question:5

The decimal expansion of the number \sqrt{2} is.
(A) A finite decimal
(B) 1.41421
(C) non-terminating recurring
(D) non-terminating non-recurring

Answer:

Answer: [D]
Solution.
Terminating decimals have a finite number of digits after decimal point,
Examples: 1/2 = 0.5, 3/5 = 0.6
Non terminating decimals are the ones which keep on continuing after decimal point.
Examples: 1/3 = 0.33333...., 5/11 = 0.454545...
Recurring decimals are those non terminating decimals which have a particular pattern/sequence that keeps on repeating itself after the decimal point. They are also called repeating decimals.
Examples: 1/3 = 0.33333..., 4/11 = 0.363636....
Non-Recurring decimals are those non terminating decimals which do not have a particular pattern/sequence after the decimal point. They are also called non repeating decimals.
Examples:
\sqrt{2}=1.414213562373
\sqrt{3}=1.732050807568
\pi =3.14159265359
So, the decimal expansion of the number \sqrt{2} is non-terminating non-recurring. It is an irrational number which is a non-terminating non-recurring decimal expansion.
Therefore option (D) is correct.

Question:6

Which of the following is irrational?
(A) \sqrt{\frac{4}{9}}
(B) \frac{\sqrt{12}}{\sqrt{3}}
(C) \sqrt{7 }
(D) \sqrt{81}

Answer:

Answer: [C]
Solution.

Irrational numbers are real numbers which cannot be represented as simple fractions.
Examples: \sqrt{2},\sqrt{3},\pi
\sqrt{\frac{4}{9}}=\frac{2}{3} (rational)
\frac{\sqrt{12}}{\sqrt{3}}=\frac{2\sqrt{3}}{\sqrt{3}}=2 (rational)
\sqrt{81}=9 (rational)
but \sqrt{7} is an irrational number.
Therefore option (C) is correct.

Question:7

Which of the following is irrational?
(A) 0.14
(B) 0.14\overline{16}
(C) 0.\overline{1416}
(D) 0.4014001400014

Answer:

Answer: [D]
Solution.

Irrational numbers are real numbers which cannot be represented as simple fractions.
Examples: \sqrt{2},\sqrt{3},\pi
Irrational cannot be expressed in form of p/q form and its decimal expansion is non- terminating non-recurring decimal expansions.
Non-Recurring non terminating decimals do not have a particular pattern/sequence after the decimal point. They are also called non repeating decimals.
Examples:
\sqrt{2}=1.414213562373
\sqrt{3}=1.732050807568
\pi =3.14159265359
In the given options, only option D is non-terminating non-recurring decimal as it satisfies the above definition.
Therefore option (D) is correct.

Question:8

A rational number between \sqrt{2} and \sqrt{3} is –
(A) \frac{\sqrt{2}+\sqrt{3}}{2}
(B) \frac{\sqrt{2}.\sqrt{3}}{2}
(C) 1.5
(D) 1.8

Answer:

Answer: [C]
Solution.

Any number which can be represented in the form of p/q where q is not equal to zero is a rational number. So it is basically a fraction with non-zero denominator.
Irrational numbers are real numbers which cannot be represented as simple fractions.
Now, we have \sqrt{2}=1.414 and \sqrt{3}= 1.732
(A) \frac{\sqrt{2}+\sqrt{3}}{2} cannot be represented as simple fraction hence it is irrational.
(B) \frac{\sqrt{2}.\sqrt{3}}{2} =\frac{\sqrt{3}}{\sqrt{2}}=\sqrt{\frac{3}{2}}=\sqrt{1.5} cannot be represented as simple fraction hence it is irrational.
(C) 1.5 is a rational number between \sqrt{2}=1.414 and \sqrt{3}= 1.732
Q 1.414 < 1.5 < 1.732
(D) 1.8 is a rational number but does not lie between
\sqrt{2}=1.414 and \sqrt{3}= 1.732.
Q 1.732 < 1.8

Therefore option (C) is correct.

Question:9

The value of 1.999 ___in the form \frac{p}{q} where p and q are integers and q \neq0, is
(A)\frac{19}{10}
(B)\frac{1999}{1000}
(C)2
(D)\frac{1}{9}

Answer:

Answer. [C]
Solution.

Let x = 1.999……
Since, one digit is repeating, we multiply x by 10
we get, 10x = 19.999……..
so, 10x = 18 + 1.999………
10x = 18 + x
Therefore, 10x – x = 18, i.e., 9x = 18
i.e., x= \frac{18}{9}= \frac{2}{1}= 2
Hence option C is correct answer.

Question:10

2\sqrt{3}+\sqrt{3} is equal to –
(A).2\sqrt{6}
(B) 6
(C)3\sqrt{3}
(D) 4\sqrt{6}

Answer:

Answer. [C]
Solution.
We can see that both the numbers \sqrt{3} and 2\sqrt{3} are irrational numbers.
So their decimal representation will be non terminating, non repeating. So we
cannot find the exact answer by adding their decimal representations.
Now 2\sqrt{3}+\sqrt{3} can be written as
= \sqrt{3}\left ( 2+1 \right ) Taking \sqrt{3} common
Hence the correct answer is 3\sqrt{3}

Question:11

\sqrt{10}\times \sqrt{15}is equal to :
(A)6\sqrt{5}
(B)5\sqrt{6}
(C)\sqrt{25}
(D)10\sqrt{5}

Answer:

Answer. [B] 5\sqrt{6}
Solution.
We know that
10= 5\times 2
15= 5\times 3
So we have,
\sqrt{10}\times \sqrt{15}= \sqrt{5\times 2}\times \sqrt{5\times 3}
This can be written as
= \sqrt{5\times 5\times2\times3}
=5 \sqrt{2\times 3}
=5 \sqrt{6}
Therefore option (B) is correct.

Question:12

The number obtained on rationalizing the denominator of \frac{1}{\sqrt{7}-2} is :
(A) \frac{\sqrt{7}+2}{3}
(B)\frac{\sqrt{7}-2}{3}
(C)\frac{\sqrt{7}+2}{5}
(D)\frac{\sqrt{7}+2}{45}

Answer:

Answer. [A]
Solution
.
We have, \frac{1}{\sqrt{7}-2}
We have to rationalize it
\frac{1}{\sqrt{7}-2}\times \frac{\sqrt{7}+2}{\sqrt{7}+2} [Multiplying numerator and denominator by \sqrt{7}+2]
= \frac{\sqrt{7}+2}{\left ( \sqrt{7} \right )^{2}-\left ( 2 \right )^{2}} [\because (a – b) (a + b) = a2 – b2]
= \frac{\sqrt{7}+2}{7-4}
= \frac{\sqrt{7}+2}{3}
Hence option A is correct.

Question:13

\frac{1}{\sqrt{9}-\sqrt{8}} is equal to :
(A) \frac{1}{2}\left ( 3-2\sqrt{2} \right )
(B) \frac{1}{3+2\sqrt{2}}
(C)3-2\sqrt{2}
(D) 3+2\sqrt{2}

Answer:

Answer.[D]
Solution.
We have, \frac{1}{\sqrt{9}-\sqrt{8}}
We have to rationalize it
\frac{1}{\sqrt{9}-\sqrt{8}}\times \frac{\sqrt{9}+\sqrt{8}}{\sqrt{9}+\sqrt{8}} [Multiplying and dividing by \sqrt{9}+\sqrt{8}]
= \frac{\sqrt{9}+\sqrt{8}}{\left ( \sqrt{9} \right )^{2}-\left ( \sqrt{8} \right )^{2}} [\because (a – b) (a + b) = a2 – b2]
=\frac{3+2\sqrt{2}}{9-8}
= 3+2\sqrt{2}
Hence option D is correct.

Question:14

After rationalizing the denominator of \frac{7}{3\sqrt{3}-2\sqrt{2}} we get the denominator as:
(A)13
(B)19
(C)5
(D)35

Answer:

Answer. [B]
Solution.

We have,\frac{7}{3\sqrt{3}-2\sqrt{2}}
We have to rationalize it
\frac{7}{3\sqrt{3}-2\sqrt{2}}\times \frac{3\sqrt{3}+2\sqrt{2}}{3\sqrt{3}+2\sqrt{2}}
[Multiplying numerator and denominator by 3\sqrt{3}+2\sqrt{2}]
= \frac{7\left ( 3\sqrt{3}+2\sqrt{2} \right )}{\left ( 3\sqrt{3} \right )^{2}-\left ( 2\sqrt{2} \right )^{2}} [\because (a – b) (a + b) = a2 – b2]
\frac{7\left ( 3\sqrt{3} \right )+2\sqrt{2}}{27-8}
= \frac{7\left ( 3\sqrt{3} \right )+2\sqrt{2}}{19}
Therefore we get the denominator as 19.
Hence (B) is the correct option.

Question:15

The value of \frac{\sqrt{32}+\sqrt{48}}{\sqrt{8}+\sqrt{12}} is equal to :
(A) \sqrt{2}
(B) 2
(C)4
(D) 8

Answer:

Answer. [B]
Solution.
\sqrt{32}+\sqrt{48}= \sqrt{16\times 2}+\sqrt{16\times3}= 4\left ( \sqrt{2}+\sqrt{3} \right )
\sqrt{8}+\sqrt{12}= \sqrt{4\times 2}+\sqrt{4\times3}= 2\left ( \sqrt{2}+\sqrt{3} \right )
\frac{\sqrt{32}+\sqrt{48}}{\sqrt{8}+\sqrt{12}}= \frac{4\left ( \sqrt{2}+\sqrt{3} \right )}{2\left ( \sqrt{2} \right )+\sqrt{3}}= 2

Hence (B) is the correrct option.

Question:16

If \sqrt{2}= 1\cdot 4142 = 1.4142 then \sqrt{\frac{\sqrt{2}-1}{\sqrt{2}+1}} is equal to :
(A)2.4142
(B)5.8282
(C)0.4142
(D)0.1718

Answer:

Answer. [C]
Solution

We have, \sqrt{\frac{\sqrt{2}-1}{\sqrt{2}+1}}
We have to rationalize it
\sqrt{\frac{\sqrt{2}-1}{\sqrt{2}+1}\times \frac{\sqrt{2}-1}{\sqrt{2}-1}} [Multiplying numerator and denominator by \sqrt{2}-1]
= \frac{\sqrt{\left ( \sqrt{2}-1 \right )\times\left ( \sqrt{2} -1\right ) }}{\sqrt{\left ( \sqrt{2} \right )^{2}-\left ( 1 \right )^{2}}} [\because (a – b) (a + b) = a2 – b2]
= \frac{\sqrt{\left ( \sqrt{2}-1 \right )^{2}}}{1}
\sqrt{\left ( \sqrt{2}-1 \right )^{2}}
=\sqrt{2}-1
=1\cdot 4142-1
=0.4142
Hence option C is correct.

Question:17

\sqrt[4]{\sqrt[3]{2^{2}}} equals :-
(A) 2^{-\frac{1}{6}}
(B)2-6
(C) 2^{\frac{1}{6}}
(D) 2^{6}

Answer:

Answer. [C]
Solution.

We have, \sqrt[4]{\sqrt[3]{2^{2}}} =\left ( \left ( 2^{2} \right )^{}\frac{1}{3} \right )^{\frac{1}{4}}
\because \sqrt[n]{a}= \left ( a \right )^{\frac{1}{n}}
\because \left ( a^{m} \right )^{n}= a^{m\times n}
So we get,
\left ( 2 \right )^{2\times \frac{1}{3}\times \frac{1}{4}}
= \left ( 2 \right )^{\frac{1}{3}\times \frac{1}{2}}
= \left ( 2 \right )^{\frac{1}{6}}
Hence option C is correct.

Question:18

The product \sqrt[3]{2}.\sqrt[4]{2}.\sqrt[12]{32} equals:
(A)\sqrt{2}
(B)2
(C)12\sqrt{2}
(D)12\sqrt{32}

Answer:

Answer. [B]
Solution.
We have, \sqrt[3]{2}.\sqrt[4]{2}.\sqrt[12]{32}
We know that \sqrt[n]{a}= \left ( a \right )^{\frac{1}{n}}
\sqrt[3]{2}.\sqrt[4]{2}.\sqrt[12]{32}=\left ( 2 \right )^{\frac{1}{3}}\left ( 2 \right )^{\frac{1}{4}}\left ( 2\cdot 2\cdot 2\cdot 2\cdot 2 \right )^{\frac{1}{12}}
= \left ( 2 \right )^{\frac{1}{3}}\left ( 2 \right )^{\frac{1}{4}}\left ( 2^{5} \right )^{\frac{1}{12}}
= \left ( 2 \right )^{\frac{1}{3}}\left ( 2 \right )^{\frac{1}{4}}\left ( 2 \right )^{\frac{5}{12}} \because \left ( a^{m} \right )^{n}= a^{m\times n}
= \left ( 2 \right )^{\frac{1}{3}+ \frac{1}{4}+\frac{5}{12}}
\because a^{m}\times a^{n}= a^{m+n}
Now, \frac{1}{3}+\frac{1}{4}+\frac{5}{12}= \frac{4+3+5}{12}= \frac{12}{12}= 1
So, \left ( 2 \right )^{\frac{1}{3}+\frac{1}{4}+\frac{5}{12}}=2^{1}= 2
Hence option B is correct.

Question:19

The value of \sqrt[4]{\left ( 81 \right )^{-2}}is :
(A)\frac{1}{9}
(B)\frac{1}{3}
(C)9
(D)\frac{1}{81}

Answer:

Answer. [A]
Solution.
We have, \sqrt[4]{\left ( 81 \right )^{-2}}
We know that \sqrt[n]{a}= \left ( a \right )^{\frac{1}{n}}
So,
\sqrt[4]{\left ( 81 \right )^{-2}}= \left ( \left ( 81 \right )^{-2} \right )^{\frac{1}{4}}
=\left ( 81 \right )^{-2\times \frac{1}{4}} \because \left ( a^{m} \right )^{n}= a^{m\times n}
= 81^{-\frac{1}{2}}= \left ( \frac{1}{81} \right )^{\frac{1}{2}} \because \left ( a^{-m}= \left ( \frac{1}{a} \right )^{m} \right )
= \sqrt{\frac{1}{81}}
=\frac{1}{9}
Hence option A is correct

Question:20

Value of (256)0.16 × (256)0.09 is :
(A) 4
(B) 16
(C) 64
(D) 256.25

Answer:

Answer. [A]
Solution.
We have,
(256)0.16 × (256)0.09 [\because am × an = am+n]
= (256)0.16 + 0.09
= (256)0.25
= \left ( 256 \right )^{\frac{25}{100}}
= \left ( 256 \right )^{\frac{1}{4}}
Now 256 = 28 = (22)4 = 44
= (44)1/4 [\because (am)n = amn]
= 4
Hence option A is correct.

Question:21

Which of the following is equal to x?
(A)x^{\frac{12}{7}}+x^{\frac{5}{7}}
(B)\sqrt[12]{\left ( x^{4} \right )^{\frac{1}{3}}}
(C)\left ( \sqrt{x^{3}} \right )^{\frac{2}{3}}
(D)x^{\frac{12}{7}}\times x^{\frac{7}{12}}

Answer:

Answer. [C]
Solution.
(A) We have,
x^{\frac{12}{7}}+x^{\frac{5}{7}}= x^{\frac{1}{7}\left ( 12 \right )}+x^{\frac{1}{7}\left ( 5 \right )}
= x^{\frac{1}{7}}\left ( x^{12} +x^{5}\right )\neq x
(B) We have,
\sqrt[12]{\left ( x^{4} \right )^{\frac{1}{3}}}=\left ( \left ( x^{4} \right ) ^{\frac{1}{3}}\right )^{\frac{1}{12}} \left ( \sqrt[n]{a} = \left ( a \right )^{\frac{1}{n}}\right )
= x^{4\times ^{\frac{1}{3}\times \frac{1}{12}}}= x^{\frac{1}{9}} \because \left ( a^{m} \right )^{n}= a^{m\times n}
\neq x
(C) We have,
\left ( \sqrt{x^{3}} \right )^{\frac{2}{3}}= \left ( \left ( x^{3} \right ) ^{\frac{2}{3}}\right )^{\frac{1}{2}} \left ( \sqrt[n]{a} = \left ( a \right )^{\frac{1}{n}}\right )
= x^{3\times \frac{2}{3}\times \frac{1}{2}} \because \left ( a^{m} \right )^{n}= a^{m\times n}
= x
(D) We have,
x^{\frac{12}{7}}\times x^{\frac{7}{12}}= x^{\frac{12}{7}+\frac{7}{12}}= x^{\frac{144+49}{84}}\neq x
\because a^{m}\times a^{n}= a^{m+n}
Hence option C is correct.

NCERT Exemplar Maths Solutions Class 9 Chapter 1: Exercise-1.2

Question:1

Let x and y be rational and irrational number respectively. Is x + y necessarily an irrational number?
Give an example in support of your answer.

Answer:

Solution.
Any number which can be represented in the form of p/q where q is not equal to zero is a rational number. So it is basically a fraction with non-zero denominator.
Irrational numbers are real numbers which cannot be represented as simple fractions.
Given that x and y be rational and irrational number respectively.
So, (x + y) is necessarily an irrational number.
For example, let x=2,y=\sqrt{3}
Then, x + y = 2+\sqrt{3}
If possible, let us assume x + y = 2+\sqrt{3} be a rational number.

Consider a = 2 + \sqrt{3}
On squaring both sides, we get
a^{2}=(2+\sqrt{3})^{2}
(using identity (a + b)^{2} = a^{2} + b^{2} + 2ab)
\Rightarrow a^{2}=2^{2}+(\sqrt{3})^{2}+2(2)(\sqrt{3})
\Rightarrow a^{2}=4+3+4\sqrt{3}
\Rightarrow \frac{a^{2}-7}{4}=\sqrt{3}
But we have assumed a is rational
\Rightarrow \frac{a^{2}-7}{4}is rational
\Rightarrow \sqrt{3} is rational which is not true.
Hence our assumption was incorrect, so 2+ \sqrt{3}is irrational.
Hence proved

Question:2

Let x be rational and y be irrational. Is xy necessarily irrational? Justify your answer by an example.

Answer:

Answer: [xy is not necessarily an irrational number.]
Solution.
Any number which can be represented in the form of p/q where q is not equal to zero is a rational number. So it is basically a fraction with non-zero denominator.
Irrational numbers are real numbers which cannot be represented as simple fractions.
Given that x and y be rational and irrational number respectively.
Let x = 0 (a rational number) and y=\sqrt{3}be an irrational number. then,
xy=0(\sqrt{3})=0, which is not an irrational number.
Hence, xy is not necessarily an irrational number.

Question:3

(i) State whether the given statement is true or false? Justify your answer by an example. \frac{\sqrt{2}}{3} is a rational number.
(ii) State whether the given statement is true or false? Justify your answer by an example. There are infinitely many integers between any two integers.
(iii) State whether the given statement is true or false? Justify your answer by an example. Number of rational numbers between 15 and 18 is finite.
(iv) State whether the given statement is true or false? Justify your answer by an example. There are numbers which cannot be written in the form \frac{p}{q} ,q \neq 0, p and q both are integers.

(v) State whether the given statement is true or false? Justify your answer by an example. The square of an irrational number is always rational.
(vi) State whether the given statement is true or false? Justify your answer by an example. \frac{\sqrt{12}}{\sqrt{3}} is not a rational number as \sqrt{12} and \sqrt{3} are not integers.
(vii) State whether the given statement is true or false? Justify your answer by an example. \frac{\sqrt{15}}{\sqrt{3}}is written in the form \frac{p}{q}, q \neq 0and so it is a rational number.

Answer:

(i)
Answer: False
Solution.
Any number which can be represented in the form of p/q where q is not equal to zero is a rational number. So it is basically a fraction with non-zero denominator.
Examples: 4/5, 2/3, -6/7
Irrational numbers are real numbers which cannot be represented as simple fractions.
Examples: \sqrt{2},\sqrt{3},\pi
The given number is \frac{\sqrt{2}}{3}.
Here \sqrt{2} is an irrational number and 3 is a rational number, we know that when we divide irrational number by non-zero rational it always gives an irrational number.
Hence the given number is irrational.
Therefore the given statement is False.

(ii)
Answer: False
Solution.
An integer is a number which can be written without fractional components or decimal representation. They can be positive, negative or zero.
Examples: \left \{ .....-4,-3,-2,-1,0,1,2,3,4,..... \right \}
The given statement is “There are infinitely many integers between any two integers.”
This is false, because between two integers (like 1 and 9), there does not exist infinite integers.
Also, if we consider two consecutive integers (like 8 and 9), there does not exist any integer between them.
Therefore the given statement is False.

(iii)
Answer: False
Solution.
Any number which can be represented in the form of p/q where q is not equal to zero is a rational number. So it is basically a fraction with non-zero denominator.
Examples: 4/5, 2/3, -6/7
So, it is a type of real number.
The given statement is: Number of rational numbers between 15 and 18 is finite.
If we see the definition of rational numbers as mentioned above, the given statement is false, because between any two rational numbers there exist infinitely many rational numbers.
Here we have rational numbers between 15 and 18 as:
16,16.1\left ( =\frac{161}{10} \right ),16.2\left ( =\frac{162}{10} \right ),16.12\left ( =\frac{1612}{100} \right ),..... and infinitely more.
Therefore the given statement is False.

(iv) Answer: True,
Solution:
There are infinitely many numbers which cannot be written in the form \frac{p}{q} ,q \neq 0, p and q both are integers. These numbers are called irrational numbers
(v)
Answer: False
Solution.
Any number which can be represented in the form of p/q where q is not equal to zero is a rational number.
Irrational numbers are real numbers which cannot be represented as simple fractions.
The given statement is: The square of an irrational number is always rational.
This is False, e.g., let us consider irrational numbers \sqrt{2} and \sqrt[4]{2}
(a)(\sqrt{2})^{2}=2,which is a rational number.
(b)(\sqrt[4]{2})^{2}=\sqrt{2}, which is an irrational number.
Hence, square of an irrational number is not always a rational number.
Therefore the given statement is False.
(vi)
Answer: False
Solution:
\frac{\sqrt{12}}{\sqrt{3}}=\frac{\sqrt{4 \times 3}}{\sqrt{3}}=\frac{\sqrt{4} \times \sqrt{3}}{\sqrt{3}}=2 \times 1=2which is a rational number.
(vii)
Answer: False
Solution:
\frac{\sqrt{15}}{\sqrt{3}}=\frac{\sqrt{5 \times 3}}{\sqrt{3}}=\frac{\sqrt{5} \times \sqrt{3}}{\sqrt{3}}=\sqrt{5}which is an irrational number.

Question:4

(i) Classify the given number as rational or irrational with justification.\sqrt{196}.
(ii) Classify the given number as rational or irrational with justification 3\sqrt{18}
(iii) Classify the given number as rational or irrational with justification.\sqrt{\frac{9}{27}}
(iv) Classify the given number as rational or irrational with justification.\frac{\sqrt{28}}{\sqrt{343}}
(v) Classify the given number as rational or irrational with justification.-\sqrt{0\cdot 4}
(vi) Classify the given number as rational or irrational with justification.\frac{\sqrt{12}}{\sqrt{75}}
(vii) Classify the given number as rational or irrational with justification.0.5918
(viii) Classify the given number as rational or irrational with justification.\left ( 1+\sqrt{5} \right )-\left ( 4+\sqrt{5} \right )
(ix) Classify the given number as rational or irrational with justification.10.124124 ………..
(x) Classify the given number as rational or irrational with justification.1.010010001 ………….

Answer:

(i)Answer. [Rational]
Solution.

We have,
\sqrt{196} = 14 = \frac{14}{1} which follows rule of rational number.
Any number which can be represented in the form of p/q where q is not equal to zero is a rational number. Also, both p and q should be rational when the fraction is expressed in the simplest form.

So, \sqrt{196} is a rational number.

(ii)Answer. [Irrational]
Solution.

Any number which can be represented in the form of p/q where q is not equal to zero is a rational number. Also, both p and q should be rational when the fraction is expressed in the simplest form.
We have,
3\sqrt{18}= 3\sqrt{9\times 2}
= 3\sqrt{9}\sqrt{2}
= 3\times 3\sqrt{2}= 9\sqrt{2}
So, it can be written in the form of \frac{p}{q} as \frac{9\sqrt{2}}{1}
But we know that 9\sqrt{2} is irrational
(Irrational numbers are real numbers which cannot be represented as simple fractions.)
Hence, 3\sqrt{18} is an irrational number
(iii)Answer. [Irrational]
Solution.

Any number which can be represented in the form of p/q where q is not equal to zero is a rational number. Also, both p and q should be rational when the fraction is expressed in the simplest form.
We have,
\sqrt{\frac{9}{27}}= \sqrt{\frac{3\times 3}{3\times 3\times 3}}
= \sqrt{\frac{1}{3}}= \frac{\sqrt{1}}{\sqrt{3}}= \frac{1}{\sqrt{3}}
So this can be written in the form of \frac{p}{q} as \frac{1}{\sqrt{3}} but we can see that \sqrt{3} (denominator) is irrational.
(Irrational numbers are real numbers which cannot be represented as simple fractions.)
Hence \sqrt{\frac{9}{27}} is irrational

(iv)Answer. [Rational]
Solution.

We have,
\frac{\sqrt{28}}{\sqrt{343}}= \frac{\sqrt{4\times 7}}{\sqrt{49\times 7}}
= \frac{2\times \sqrt{7}}{7\times \sqrt{7}}= \frac{2}{7}
Any number which can be represented in the form of p/q where q is not equal to zero is a rational number. Also, both p and q should be rational when the fraction is expressed in the simplest form.
Hence \frac{\sqrt{28}}{\sqrt{343}} is a rational number.
(v)Answer. [Irrational]
Solution.

Any number which can be represented in the form of p/q where q is not equal to zero is a rational number. Also, both p and q should be rational when the fraction is expressed in the simplest form.
We have,
-\sqrt{0\cdot 4}= -\sqrt{\frac{4}{10}}
= -\sqrt{\frac{2}{5}}= -\frac{\sqrt{2}}{\sqrt{5}}
So, it can be written in the form of \frac{p}{q} as \frac{-\sqrt{2}}{\sqrt{5}}
But we know that both \sqrt{2},\sqrt{5} are irrational
(Irrational numbers are real numbers which cannot be represented as simple fractions.)
Hence, -\sqrt{0\cdot 4} is an irrational number

(vi)Answer. [Rational]
Solution.
We have,
\frac{\sqrt{12}}{\sqrt{75}}= \frac{\sqrt{4\times 3}}{\sqrt{25\times 3}}
= \frac{\sqrt{4}\sqrt{3}}{\sqrt{25}\sqrt{3}}= \frac{\sqrt{4}}{\sqrt{25}}
= \frac{2}{5}
Any number which can be represented in the form of p/q where q is not equal to zero is a rational number. Also, both p and q
should be rational when the fraction is expressed in the simplest form.
So, \frac{\sqrt{12}}{\sqrt{75}} is a rational number.
(vii)Answer. [Rational]
Solution.

We have,
0\cdot 5918= \frac{0\cdot 5918\times 10000}{1\times 10000}
= \frac{5918}{10000}= \frac{2959}{5000}
Any number which can be represented in the form of p/q where q is not equal to zero is a rational number. Also, both p and q should be rational when the fraction is expressed in the simplest form.
Also we can see that 0.5918 is a terminating decimal number hence it must be rational.
So, 0.5918 is a rational number.
(viii)Answer. [Rational]
Solution.

We have,
\left ( 1+\sqrt{5} \right )-\left ( 4+\sqrt{5} \right )
= 1+\sqrt{5}-4-\sqrt{5}
= -3= \frac{-3}{1}
Any number which can be represented in the form of p/q where q is not equal to zero is a rational number. Also, both p and q should be rational when the fraction is expressed in the simplest form.
So, \left ( 1+\sqrt{5} \right )-\left ( 4+\sqrt{5} \right ) is a rational number.

(ix)Answer. [Rational]
Solution.

Any number which can be represented in the form of p/q where q is not equal to zero is a rational number. Also, both p and q should be rational when the fraction is expressed in the simplest form.
All non-terminating recurring decimal numbers are rational numbers.
Non terminating Recurring decimals are those decimals which have a particular pattern/sequence that keeps on repeating
itself after the decimal point. They are also called repeating decimals.
Examples: 1/3 = 0.33333…, 4/11 = 0.363636…
Now, 10.124124 ………. is a decimal expansion which is a non-terminating recurring.
So, it is a rational number.
(x)Answer. [Irrational]
Solution
.
Non terminating Recurring decimals are those decimals which have a particular pattern/sequence that keeps on repeating itself after the decimal point.
All non-terminating recurring decimal numbers are rational numbers.
Non terminating Non Recurring decimals are those decimals which do not have a particular pattern/sequence after the decimal point and it does not end.
All non-terminating non-recurring decimal numbers are irrational numbers.
1.010010001 ………. is non-terminating non-recurring decimal number, therefore it cannot be written in the form \frac{p}{q};q\neq 0,with p,q both as integers.
Thus, 1.010010001 ……….. is an irrational number.

NCERT Exemplar Class 9 Maths Solutions Chapter-1: Exercise-1.3

Question:1

(i)Find whether variable x represent rational number or an irrational number.x2 = 5
(ii)Find whether variable x represent rational number or an irrational number.y2 = 9
(iii)Find whether variable x represent rational number or an irrational number.z2 = 0.04
(iv)Find whether variable x represent rational number or an irrational number.u2 = \frac{17}{4}

Answer:

(i)Answer. [Irrational]
Solution.

Given that
x2 = 5
On taking square root on both sides, we get
\Rightarrow x= \pm \sqrt{5}
Irrational numbers are real numbers which cannot be represented as simple fractions.
So, x is an irrational number

(ii)Answer. [Rational]
Solution.

We have
y2 = 9
On taking square root on both sides, we get
\Rightarrow x= \pm 3
Any number which can be represented in the form of p/q where q is not equal to zero is a rational number. Also, both p and q
should be rational when the fraction is expressed in the simplest form.
So, y can be written as \frac{3}{1},\frac{-3}{1} where 1, 3, -3 are rational numbers.
Hence y is rational.
(iii)Answer. [Rational]
Solution.

We have
z2 = 0.04 =\frac{4}{100}
On taking square root on both sides, we get
\Rightarrow z= \pm \frac{2}{10}= \pm \frac{1}{5}
Any number which can be represented in the form of p/q where q is not equal to zero is a rational number. Also, both p and q
should be rational when the fraction is expressed in the simplest form.
So, z can be written as \pm \frac{1}{5} where 1, 5, -5 are rational numbers.
Hence z is rational.
(iv)Answer. [Irrational]
Solution.

Given that
u2 = \frac{17}{4}
On taking square root on both sides, we get
\Rightarrow u= \sqrt{\frac{17}{4}}= \pm \frac{\sqrt{17}}{2}
Any number which can be represented in the form of p/q where q is not equal to zero
is a rational number. Also, both p and q should be rational when the fraction is expressed in the simplest form.
So, u can be written as \pm \frac{\sqrt{17}}{2} where \sqrt{17} is irrational.
Hence u is irrational.

Question:2

(i) Find three rational numbers between – 1 and – 2
(ii) Find three rational numbers between 0.1 and 0.11
(iii)Find three rational numbers between \frac{5}{7} and \frac{6}{7}
(iv)Find three rational numbers between \frac{1}{4} and \frac{1}{5}

Answer:

(i)Answer.
-\frac{11}{10}, -\frac{6}{5}
and-\frac{5}{4}
Solution.

Any number which can be represented in the form of p/q where q is not equal to zero is a rational number. Also, both p and q should be rational when the fraction is expressed in the simplest form.

Between -1 and -2, many rational number can be written as:
-1\cdot 1=-\frac{11}{10}
-1\cdot 2=-\frac{12}{10}= -\frac{6}{5}
-1\cdot 25=-\frac{125}{100}= -\frac{5}{4}
-1\cdot 3=-\frac{13}{10}
-1\cdot 4=-\frac{14}{10}= -\frac{7}{5}
(ii) Answer: \frac{103}{1000},\frac{104}{1000},\frac{105}{1000}
Solution.
Any number which can be represented in the form of p/q where q is not equal to zero is a rational number. Also, both p and q should be rational when the fraction is expressed in the simplest form.

Between 0.1 and 0.11, many rational number can be written as:

0.103 = \frac{103}{1000}

0.104 = \frac{104}{1000}

0.105 = \frac{105}{1000}


(iii)Answer. \frac{51}{70},\frac{52}{70}and\frac{53}{70}
Solution.

Any number which can be represented in the form of p/q where q is not equal to zero is a rational number. Also, both p and q should be rational when the fraction is expressed in the simplest form.
We can write \frac{5}{7}as \frac{5\times 10}{7\times 10}= \frac{50}{70} and \frac{6}{7} as \frac{6\times 10}{7\times 10}= \frac{60}{70}
So, three rational number between \frac{5}{7}and\frac{6}{7} are \frac{51}{70},\frac{52}{70}and\frac{53}{70}

(iv)Answer. \frac{41}{200},\frac{42}{200},\frac{43}{200}
Solution.
Any number which can be represented in the form of p/q where q is not equal to zero is a rational number. Also, both p and q should be rational when the fraction is expressed in the simplest form.
L.C.M. of 4 and 5 is 20.
We can write \frac{1}{4} as \frac{1\times 40}{4\times 50}= \frac{50}{200}
and \frac{1}{5}\, as\, \frac{1\times 40}{5\times 40}= \frac{40}{200}
So, three rational number between \frac{1}{4}\, and\,\frac{1}{5} are
\frac{41}{200},\frac{42}{200},\frac{43}{200}

Question:3

(i)Insert a rational number and an irrational number between the following. 2 and 3
(ii)Insert a rational number and an irrational number between the following.0.1 and 0.1
(iii)Insert a rational number and an irrational number between the following.\frac{1}{3}\, and\, \frac{1}{2}
(iv)Insert a rational number and an irrational number between the following.\frac{-2}{5}\, and\, \frac{1}{2}
(v)Insert a rational number and an irrational number between the following.0.15 and 0.16
(vi)Insert a rational number and an irrational number between the following.\sqrt{2} and \sqrt{3}
(vii)Insert a rational number and an irrational number between the following.2.357 and 3.121
(viii)Insert a rational number and an irrational number between the following..0001 and .001
(ix)Insert a rational number and an irrational number between the following.3.623623 and 0.484848
(x) Insert a rational number and an irrational number between the following.6.375289 and 6.375738

Answer:

(i) Answer. Rational number: \frac{5}{2}
Irrational number: 2.040040004 ……….
Solution.
Any number which can be represented in the form of p/q where q is not equal to zero is a rational number. Also, both p and q should be rational when the fraction is expressed in the simplest form.
Irrational numbers are real numbers which cannot be represented as simple fractions.
They are non-terminating non-recurring in nature. It means they keep on continuing after decimal point and do not have a particular pattern/sequence after the decimal point.
Between 2 and 3
Rational number: 2.5 = \frac{25}{10}= \frac{5}{2}
and irrational number : 2.040040004

(ii) Answer. Rational number: \frac{19}{1000}
Irrational number 0.0105000500005 ……..
Solution.
Any number which can be represented in the form of p/q where q is not equal to zero is a rational number. Also, both p and q should be rational when the fraction is expressed in the simplest form.
Irrational numbers are real numbers which cannot be represented as simple fractions.
They are non-terminating non-recurring in nature. It means they keep on continuing after decimal point and do not have a particular pattern/sequence after the decimal point.
Between 0 and 0.1:
0.1 can be written as 0.10
Rational number: 0.019 = \frac{19}{1000}
and irrational number 0.0105000500005

(iii)Answer. Rational number \frac{21}{60}
Irrational number : 0.414114111 ……
Solution.
Any number which can be represented in the form of p/q where q is not equal to zero is a rational number. Also, both p and q should be rational when the fraction is expressed in the simplest form.
Irrational numbers are real numbers which cannot be represented as simple fractions. They are non-terminating non-recurring in nature. It means they keep on continuing after decimal point and do not have a particular pattern/sequence after the decimal point.
LCM of 3 and 2 is 6.
We can write \frac{1}{3} as \frac{1\times 20}{3\times 20}= \frac{20}{60}
and \frac{1}{2} as \frac{1\times 30}{3\times 30}= \frac{30}{60}
Also, \frac{1}{3} = 0.333333….
And \frac{1}{2}= 0\cdot 5
So, rational number between \frac{1}{3} and \frac{1}{2} is \frac{21}{60}
and irrational number : 0.414114111 ……

(iv)Answer. Rational number: 0
Irrational number: 0.151551555 …….
Solution.
Any number which can be represented in the form of p/q where q is not equal to zero is a rational number. Also, both p and q should be rational when the fraction is expressed in the simplest form.
Irrational numbers are real numbers which cannot be represented as simple fractions.
They are non-terminating non-recurring in nature. It means they keep on continuing after decimal point and do not have a particular pattern/sequence after the decimal point.
\frac{-2}{5}= -0\cdot 4 and \frac{1}{2}= -0\cdot 5
Rational number between -0.4 and 0.5 is 0
And irrational number: 0.151551555 …….
(v) Answer. Rational number: \frac{151}{1000}
Irrational number: 0.151551555 ……….
Solution.
Any number which can be represented in the form of p/q where q is not equal to zero is a rational number. Also, both p and q should be rational when the fraction is expressed in the simplest form.
Irrational numbers are real numbers which cannot be represented as simple fractions.
They are non-terminating non-recurring in nature. It means they keep on continuing after decimal point and do not have a particular pattern/sequence after the decimal point.
Between 0.15 and 0.16
Rational number : 0.151 = \frac{151}{1000}
and irrational number 0.151551555

(vi) Answer. Rational number: \frac{3}{2}
Irrational number: 1.585585558 ………
Solution.
Any number which can be represented in the form of p/q where q is not equal to zero is a rational number. Also, both p and q should be rational when the fraction is expressed in the simplest form.
Irrational numbers are real numbers which cannot be represented as simple fractions. They are non-terminating non-recurring in nature.
Between \sqrt{2}\, and\, \sqrt{3}
\sqrt{2}= 1\cdot 414213562373
\sqrt{3}= 1\cdot 732050807568
Rational number: 1.5 = \frac{3}{2}
and irrational number: 1.585585558

(vii) Answer. Rational number: 3
Irrational number: 3.101101110………
Solution.
Any number which can be represented in the form of p/q where q is not equal to zero is a rational number. Also, both p and q should be rational when the fraction is expressed in the simplest form.
Irrational numbers are real numbers which cannot be represented as simple fractions.
They are non-terminating non-recurring in nature. It means they keep on continuing after decimal point and do not have a particular pattern/sequence after the decimal point.
Between 2.357 and 3.121
Rational number: 3
Irrational number: 3.101101110………
(viii) Answer. Rational number: \frac{2}{10000}
Irrational number: 0.000113133133 ……….
Solution.
Any number which can be represented in the form of p/q where q is not equal to zero is a rational number. Also, both p and q should be rational when the fraction is expressed in the simplest form.
Irrational numbers are real numbers which cannot be represented as simple fractions.
They are non-terminating non-recurring in nature. It means they keep on continuing after decimal point and do not have a particular pattern/sequence after the decimal point.
Between 0.0001 and 0.001
Rational number: 0.0002 = \frac{2}{10000}
Irrational number: 0.000113133133

(ix) Answer. Rational number: 1
Irrational number: 1.909009000 ……
Solution.
Any number which can be represented in the form of p/q where q is not equal to zero is a rational number. Also, both p and q should be rational when the fraction is expressed in the simplest form.
Irrational numbers are real numbers which cannot be represented as simple fractions.
They are non-terminating non-recurring in nature. It means they keep on continuing after decimal point and do not have a particular pattern/sequence after the decimal point.
Between 3.623623 and 0.484848
A rational number between 3.623623 and 0.484848 is 1.
An irrational number between 3.623623 and 0.484848 is 1.909009000 ……

(x) Answer. A rational number is \frac{63753}{10000}
An irrational number is 6.375414114111……..
Solution.
Any number which can be represented in the form of p/q where q is not equal to zero is a rational number. Also, both p and q should be rational when the fraction is expressed in the simplest form.
Irrational numbers are real numbers which cannot be represented as simple fractions.
They are non-terminating non-recurring in nature. It means they keep on continuing after decimal point and do not have a particular pattern/sequence after the decimal point.
Between 6.375289 and 6.375738:
A rational number is 6.3753 = \frac{63753}{10000}
An irrational number is 6.375414114111……..

Question:4

Represent the following numbers on the number line. 7, 7.2, -\frac{3}{2},-\frac{12}{5}

Answer:

Solution.
Firstly we draw a number line whose mid-point is O. Mark positive numbers on right hand side of O and negative numbers on left hand side of O.
FireShot%20Capture%20003%20-%20Careers360%20CMS%20-%20cms-article%20-%20learn(i) Number 7 is a positive number. So we mark a number 7 on the right
hand side of O, which is at a 7 units distance form zero.
(ii) Number 7.2 is a positive number. So, we mark a number 7.2 on the right hand side of O, which is 7.2 units distance from zero.
(iii) Number -\frac{3}{2} or – 1.5 is a negative number so, we mark a number 1.5 on the left hand side of zero, which is at 1.5 units distance from zero.
(iv) Number -\frac{12}{5} or –2.4 is a negative number. So, we mark a number 2.4 on the left hand side of zero, which is at 2.4 units distance from zero.

Question:5

Locate \sqrt{5},\sqrt{10} and \sqrt{17} on the number line

Answer:

Hint.
Solution.

Step I- Draw number line shown in the figure.
Let the point O represent 0 (zero) and point A represent 2 units from O.
58373
Step II- Draw perpendicular AX from A on the number line and cut off arc AB = 1 unit
We have OA = 2 units and AB = 1 unit
Using Pythagoras theorem, we have.
OB2 = OA2 + AB2
OB2 = (2)2 + (1)2 = 5
OB = \sqrt{5}
Taking O as the centre and OB = \sqrt{5} as radius draw an arc cutting the line at C.
Clearly, OC = OB = \sqrt{5}.
Hence, C represents \sqrt{5} on the number line.
Similarly for \sqrt{10} and \sqrt{17}we can plot the points as follows:
\sqrt{10}= \sqrt{3^{2}+1^{2}}
\sqrt{17}= \sqrt{4^{2}+1^{2}}
583731
583732

Question:6

(i) Represent geometrically the following numbers on the number line : \sqrt{4\cdot 5}
(ii) Represent geometrically the following numbers on the number line : \sqrt{5\cdot 6}
(iii) Presentation of \sqrt{8\cdot 1} on number line :
(iv) Presentation of \sqrt{2\cdot 3} on number line:

Answer:

(i) Solution. AB = 4.5 units, BC = 1 unit
58395
OC = OD = \frac{5\cdot 5}{2} = 2.75 units
OD2 = OB2 + BD2
\left ( \frac{4\cdot 5}{2} \right )^{2}= \left ( \frac{4\cdot 5}{2} -1\right )^{2}+\left ( BD \right )^{2}
\Rightarrow BD^{2}= \left ( \frac{4\cdot 5+1}{2} \right )^{2}- \left ( \frac{4\cdot 5-1}{2} \right )^{2}
\Rightarrow BD^{2}= 4. 5
\Rightarrow BD= \sqrt{4. 5}
So the length of BD will be the required one so mark an arc of length BD on number line, this will result in the required length.

(ii) Solution. Presentation of \sqrt{5. 6}on number line.
Mark the distance 5.6 units from a fixed point A on a given line to obtain a point B such that AB = 5.6 units. From B mark a distance of 1 unit and mark a new point C. Find the mid point of AC and mark that point as O. Draw a semicircle with center O and radius OC. Draw a line
perpendicular to AC passing through B and intersecting the semicircle at O. Then BD = \sqrt{5\cdot 6}

583951
(iii) Solution
Mark the distance 8.1 units from a fixed point A on a given line to obtain a point B such that AB = 8.1 units. From B mark a distance of 1 unit and mark the new point AB. Find the mid point of AC and mark a point as O. Draw a semi circle with point O and radius OC. Draw a line perpendicular to AC passing through B and intersecting
the semicircle at D. Then BD -
\sqrt{8. 1}
583952
(iv) Solution
Mark the distance 2.3 unit from a fixed point A on a given line. To obtain a point B such that AB = 2.3 units. From B mark a distance of 1 unit and mark a new point as C. Find the mid point of AC and mark the point asO. Draw a line perpendicular to AC passing through B and intersecting the semicircle at D. Then BD = \sqrt{2. 3}
583953

Question:7

(i) Express the following in the form \frac{p}{q}, where p and q are integers and q\neq 0. 0.2
(ii) Express the following in the form \frac{p}{q}, where p and q are integers and q\neq 0.
0.888…….
(iii) Express the following in the form \frac{p}{q}, where p and q are integers and q\neq 0.5.\bar{2}
(iv) Express the following in the form \frac{p}{q}, where p and q are integers and q\neq 0.0\cdot \overline{001}
(v) Express the following in the form \frac{p}{q}, where p and q are integers and q\neq 0.0.2555……
(vii) Express the following in the form \frac{p}{q}, where p and q are integers and q\neq 0..00323232…..
(viii) Express the following in the form \frac{p}{q}, where p and q are integers and q\neq 0..404040……..

Answer:

(i) Answer.\frac{1}{5}
Solution. We know that
0.2 can be written as \frac{2}{10}
Now,
\frac{2}{10}= \frac{1}{5}
Hence the answer is \frac{1}{5}

(ii) Answer.\frac{8}{9}
Solution. Let x = 0.888….. .…(i)
Multiply RHS and LHS by 10
10 x = 8.88……. …(ii)
Subtracting equation (i) from (ii)
We get
10x – x = 8.8 – 0.8
\Rightarrow 9x= 8
\Rightarrow x= \frac{8}{9}
Hence answer is \frac{8}{9}

(iii) Answer.\frac{47}{9}
Solution. Let x = 5\cdot \bar{2} …eq. (1)
Multiply by 10 on both sides
10x = 52\cdot \bar{2} …eq (2)
Subtracting equation (1) from (2)
We get
10x – x = 52\cdot \bar{2}5\cdot \bar{2}
\Rightarrow 9x = 47
\Rightarrow x = \frac{47}{9}
Hence the answer is \frac{47}{9}

(iv) Answer.\frac{1}{999}
Solution. Let x = 0\cdot \overline{001} …. Eq. (1)
Multiply by 1000 on both sides
1000 x = 1\cdot \overline{001} …eq.(2)
Subtracting eq. (1) from (2)
We get
1000 x – x = 1\cdot \overline{001}1\cdot \overline{001}
\Rightarrow 999x = 1
\Rightarrow x = \frac{1}{999}
Hence the answer is \frac{1}{999}
(v) Answer.\frac{23}{90}
Solution. Let x = 0.2555 ….. …eq.(1)
Multiply by 10 on both sides
10x = 2.555… …eq.(2)
Multiply by 100 on both sides
100x = 25.55… …eq. (3)
Subtracting eq. (2) from (3),
We get
100x – 10x = 25.555… – 2.555…
\Rightarrow 90x = 23
\Rightarrow x = \frac{23}{90}
Hence the answer is \frac{23}{90}

(vii) Answer.\frac{8}{2475}
Solution. Let x = 0.00323232….. …eq.(1)
Multiply with 100 on both sides.
We get
100x = 0.3232… …eq(2)
Multiply again with 100 on both sides,
10000x = 32.3232… …eq(3)
Equation (3) – (2)
we get,
10000 x – 100x = 32.3232… – 0.3232…
\Rightarrow 9900x = 32
\Rightarrow x = \frac{32}{9900}
x = \frac{8}{2475}
Hence the answer is \frac{8}{2475}

(viii) Answer.\frac{40}{99}
Solution. Let x = 0.404040……. …(1)
Multiplying by 100 on both sides
we get
100x = 40.40… …(2)
Subtracting equation (1) from (2)
we get
100x – x = 40.40 – 0.40
\Rightarrow99x = 40
\Rightarrow x = \frac{40}{99}
Hence the answer is \frac{40}{99}

Question:8

Show that 0.142857142857 ……. = \frac{1}{7}

Answer:

Solution. Let, x = 0.142857142857… …(i)
Multiply (i) by 1000000, we get
1000000 x = 142857.142857… …(ii)
Subtracting equation (i) from (ii), we get
1000000 x – x = 142857.142857… - 0.142857…
999999 x = 142857
x= \frac{142857}{999999}= \frac{47619}{333333}= \frac{15873}{111111}= \frac{5291}{37037}
= \frac{481}{3367}= \frac{1}{7}
Hence, 0.142857 ……. \frac{1}{7}
Hence proved

Question:9

(i) Simplify the following : \sqrt{45}-3\sqrt{20}+4\sqrt{5}
(ii) Simplify the following : \frac{\sqrt{24}}{8}+\frac{\sqrt{54}}{9}
(iii) Simplify the following : 4\sqrt{12}\times 7\sqrt{6}
(iv) Simplify the following : 4\sqrt{28}\div 3\sqrt{7}\div 3\sqrt{7}
(v) Simplify the following : 3\sqrt{3}+2\sqrt{27}+\frac{7}{\sqrt{3}}
(vi) Simplify the following : \left ( \sqrt{3}-\sqrt{2} \right )^{2}
(vii) Simplify the following : \sqrt[4]{81}- 8\sqrt[3]{216}+15\sqrt[5]{32}+\sqrt{255}
(viii) Simplify the following : \frac{3}{\sqrt{8}}+\frac{1}{\sqrt{2}}
(ix) Simplify the following : \frac{2\sqrt{3}}{3}-\frac{\sqrt{3}}{6}

Answer:

(i) Answer.\sqrt{5}
Solution. \sqrt{45}-3\sqrt{20}+4\sqrt{5}
We know that,
45 = 3\times 3\times 5
20 = 2\times 2\times 5
So we get
\sqrt{3\times3\times5 }-3\sqrt{2\times2\times5}+4\sqrt{5}
= 3\sqrt{5}-3\left ( 2\sqrt{5} \right )+4\sqrt{5}
= 3\sqrt{5}-6\sqrt{5}+4\sqrt{5}
= 7\sqrt{5}-6\sqrt{5}
= \sqrt{5}
Hence the answer is \sqrt{5}

(ii) Answer. \frac{7\sqrt{6}}{12}
Solution. We have, \frac{\sqrt{24}}{8}+\frac{\sqrt{54}}{9}
We know that,
24= 6\times 4= 3\times 2\times 2\times 2
54= 9\times 6= 3\times 3\times 3\times 2
So we get
\frac{\sqrt{24}}{8}+\frac{\sqrt{54}}{9}= \frac{2\sqrt{6}}{8}+\frac{3\sqrt{6}}{9}
= \frac{\sqrt{6}}{4}+\frac{\sqrt{6}}{3}
Taking LCM (3,4) = 12
= \frac{3\sqrt{6}+4\sqrt{6}}{12}
= \frac{7\sqrt{6}}{12}
(iii) Answer. \sqrt[28]{2^{18} \times 3^{11}}
Solution. We have
\sqrt[4]{12}\times \sqrt[7]{6}
We know that
12 = 2\times 2\times 3
6 = 2\times 3
So we get,
=\sqrt[4]{2\times 2\times 3}\times \sqrt[7]{2\times 3}
=2^{1 / 4} \cdot 2^{1 / 4} \cdot 3^{1 / 4} \cdot 2^{1 / 7} \cdot 3^{1 / 7}
=2^{\frac{1}{4}+\frac{1}{4}+\frac{1}{7}} \times 3^{\frac{1}{4}+\frac{1}{7}}
=2^{9 / 14} \times 3^{11 / 28}
=\sqrt[28]{2^{18} \times 3^{11}}
Hence the number is \sqrt[28]{2^{18} \times 3^{11}}.

(iv)Answer. \frac{8}{\left ( 3\times \sqrt[3]{7} \right )}
Solution. We have, 4\sqrt{28}\div 3\sqrt{7}\div \sqrt[3]{7}
We know that
28 = 4\times 7
So we can write,
4\sqrt{28}\div 3\sqrt{7}\div \sqrt[3]{7}= \left [ \frac{4\sqrt{28}}{3\sqrt{7}} \right ]\div 7^{\frac{1}{3}}
= \left [ \frac{4\sqrt{4\times 7}}{3\sqrt{7}} \right ]\div 7^{\frac{1}{3}}
= \left [ \frac{4\times 2\sqrt{ 7}}{3\sqrt{7}} \right ]\div 7^{\frac{1}{3}}
= \frac{8}{3}\div 7^{\frac{1}{3}}
= \frac{8}{\left ( 3\times 7^{\frac{1}{3}} \right )}
= \frac{8}{\left ( 3\times \sqrt[3]{7} \right )}
Hence the answer is \frac{8}{\left ( 3\times \sqrt[3]{7} \right )}

(v) Answer.3\sqrt{3}+2\sqrt{27}+\frac{7}{\sqrt{3}}
We know that
27 = 3\times 3\times 3
So, 3\sqrt{3}+2\sqrt{27}+\frac{7}{\sqrt{3}} = 3\sqrt{3}+2\sqrt{3\times 3\times 3}+\frac{7}{\sqrt{3}}

= 3\sqrt{3}+2\left ( 3\sqrt{3} \right )+\frac{7}{\sqrt{3}}\times \frac{\sqrt{3}}{\sqrt{3}}
(Rationalising the denominator)
= 3\sqrt{3}+6\left ( \sqrt{3} \right )+\frac{7\sqrt{3}}{3}

= \left ( 3+6+\frac{7}{3} \right )\sqrt{3} (Taking \sqrt{3} common)
Now LCM (1,1,3) = 3
= \left ( \frac{9+18+7}{3} \right )\sqrt{3}
= \frac{34}{3}\sqrt{3}
= 19\cdot 63
Hence the answer is 19.63
(vi) Answer.5-2\sqrt{6}
Solution. Given, \left ( \sqrt{3}-\sqrt{2} \right )^{2}
We know that (a + b)2 = a2 – 2ab + b2
Comparing the given equation with the identity, we get:
\left ( \sqrt{3}-\sqrt{2} \right )^{2}= \left ( \sqrt{3} \right )^{2}-2\left ( \sqrt{3} \right )\left ( \sqrt{2} \right )+\left ( \sqrt{2} \right )^2
= 3 + 2 – 2\sqrt{3\times 2}
= 5-2\sqrt{6}
Hence the answer is 5-2\sqrt{6}

(vii) Answer. 0
Solution. We have, \sqrt[4]{81}-8 \sqrt[3]{216}+15\sqrt[5]{32}+\sqrt{225}
We know that
81 = 3\times 3\times3\times3
216 = 6\times 6\times6
32 = 2\times 2\times2\times2\times2
225 = 15\times 15
So,\sqrt[4]{81}-8 \sqrt[3]{216}+15\sqrt[5]{32}+\sqrt{225}
= \sqrt[4]{3\times3\times3\times3 }-8\sqrt[3]{6\times6\times6}+15\sqrt[5]{2\times2\times2\times2\times2}+\sqrt{15\times15}
= 3 – 8 × 6 + 15 × 2 + 15
= 3 – 48 + 30 + 15
= – 45 + 45
= 0
Hence the answer is 0

(viii) Answer. \frac{5}{2\sqrt{2}}
Solution. We have, \frac{3}{\sqrt{8}}+\frac{1}{\sqrt{2}}
We know that, 8 =2\times 2\times 2
So,
\frac{3}{\sqrt{8}}+\frac{1}{\sqrt{2}} = \frac{3}{\sqrt{2\times 2\times 2}}+\frac{1}{\sqrt{2}}
= \frac{3}{2\sqrt{2}}+\frac{1}{\sqrt{2}}
= \frac{3}{2\sqrt{2}}+\frac{2}{2\sqrt{2}}
= \frac{5}{2\sqrt{2}}
Hence the answer is \frac{5}{2\sqrt{2}}

(ix) Answer. \frac{\sqrt{3}}{2}
Solution. We have, \frac{2\sqrt{3}}{3}-\frac{\sqrt{3}}{6}
LCM (3,6) = 6
\frac{2\sqrt{3}}{3}-\frac{\sqrt{3}}{6}= \frac{4\sqrt{3}}{6}-\frac{\sqrt{3}}{6}
= \frac{4\sqrt{3}-\sqrt{3}}{6}
= \frac{3\sqrt{3}}{6}
= \frac{\sqrt{3}}{2}
Hence the answer is \frac{\sqrt{3}}{2}.

Question:10

(i) Rationalise the denominator of the following : \frac{2}{3\sqrt{3}}
(ii)Rationalise the denominator of the following : \frac{\sqrt{40}}{\sqrt{3}}
(iii) Rationalise the denominator of the following : \frac{3+\sqrt{2}}{4\sqrt{2}}
(iv)Rationalise the denominator of the following :\frac{16}{\sqrt{41}-5}
(v) Rationalise the denominator of the following : \frac{2+\sqrt{3}}{2-\sqrt{3}}
(vi) Rationalise the denominator of the following : \frac{\sqrt{6}}{\sqrt{2}+\sqrt{3}}
(vii) Rationalise the denominator of the following : \frac{\sqrt{3}+\sqrt{2}}{\sqrt{3}-\sqrt{2}}
(viii) Rationalise the denominator of the following : \frac{3 \sqrt{5}+\sqrt{3}}{\sqrt{5}-\sqrt{3}}
(ix) Rationalise the denominator of the following : \frac{4 \sqrt{3}+5 \sqrt{2}}{\sqrt{48}+\sqrt{18}}

Answer:

(i) Answer.\frac{2\sqrt{3}}{9}
Solution. We have, \frac{2}{3\sqrt{3}}
Rationalising the denominator, we get:
\frac{2}{3\sqrt{3}}\times \frac{\sqrt{3}}{\sqrt{3}}
= \frac{2\sqrt{3}}{3\sqrt{3}\sqrt{3}}
= \frac{2\sqrt{3}}{9}
Hence the answer is \frac{2\sqrt{3}}{9}

(ii) Answer.\frac{2\sqrt{30}}{3}
Solution. We have ,\frac{\sqrt{40}}{\sqrt{3}}
We know that, 40 = (2) (2) (10)
\frac{\sqrt{40}}{\sqrt{3}}= \frac{\sqrt{2\cdot 2\cdot 10}}{\sqrt{3}}= \frac{2\sqrt{10}}{\sqrt{3}}
Rationalising the denominator, we get:
= \frac{2\sqrt{10}}{\sqrt{3}}\times \frac{\sqrt{3}}{\sqrt{3}}
= \frac{2\sqrt{10}\sqrt{3}}{\sqrt{3}\sqrt{3}}
= \frac{2\sqrt{30}}{3}
Hence the answer is: \frac{2\sqrt{30}}{3}

(iii) Answer. \frac{3\sqrt{2}+2}{8}
Solution. We have \frac{3+\sqrt{2}}{4\sqrt{2}}
Rationalising the denominator, we get:
\frac{3+\sqrt{2}}{4\sqrt{2}}\times \frac{\sqrt{2}}{\sqrt{2}}
= \frac{\left ( 3+\sqrt{2} \right )\sqrt{2}}{4\sqrt{2}\sqrt{2}}
= \frac{3\sqrt{2}+2}{8}
Hence the answer is \frac{3\sqrt{2}+2}{8}

(iv) Answer. \sqrt{41}+5
Solution. We have \frac{16}{\sqrt{41}-5}
Rationalising the denominator, we get:
\frac{16}{\sqrt{41}-5}\times \frac{\sqrt{41}+5}{\sqrt{41}+5}
= \frac{16\left ( \sqrt{41}+5 \right )}{\left ( \sqrt{41}-5 \right )\sqrt{41}+5}
Using the identity (a – b) (a + b) = a2 – b2
We get:
= \frac{16\left ( \sqrt{41}+5 \right )}{\left ( \sqrt{41} \right )^{2}-\left ( 5 \right )^{2}}
= \frac{16\left ( \sqrt{41}+5 \right )}{41-25}
= \frac{16\left ( \sqrt{41}+5 \right )}{16}
= \sqrt{41}+5
Hence the answer is \sqrt{41}+5

(v) Answer.7+4\sqrt{3}
Solution. We have, \frac{2+\sqrt{3}}{2-\sqrt{3}}
Rationalising the denominator, we get:
\frac{2+\sqrt{3}}{2-\sqrt{3}}\times \frac{2+\sqrt{3}}{2-\sqrt{3}}
= \frac{\left ( 2+\sqrt{3} \right )^{2}}{\left ( 2-\sqrt{3} \right )\left ( 2+\sqrt{3} \right )}
Using (a – b) (a + b) = a2 – b2
and (a + b)2 = a2 + b2 + 2ab
= \frac{2^{2}+\left ( \sqrt{3} \right )^{2}+2\cdot 2\cdot \sqrt{3}}{2^{2}-\left ( \sqrt{3} \right )^{2}}
= \frac{4+3+4\sqrt{3}}{4-3}
= 7+4\sqrt{3}
Hence the answer is 7+4\sqrt{3}
(vi)Answer. 3\sqrt{2}-2\sqrt{3}
Solution. We have, \frac{\sqrt{6}}{\sqrt{2}+\sqrt{3}}
Rationalising the denominator, we get:
= \frac{\sqrt{6}}{\sqrt{2}+\sqrt{3}}\times \frac{\sqrt{3}-\sqrt{2}}{\sqrt{3}-\sqrt{2}}
= \frac{\sqrt{6}\left ( \sqrt{3}-\sqrt{2} \right )}{\left ( \sqrt{2}+\sqrt{3} \right )\left ( \sqrt{3}-\sqrt{2} \right )}
Using the identity (a – b) (a + b) = a2 – b2
We get:
= \frac{\sqrt{18}-\sqrt{12}}{\left ( \sqrt{3} \right )^{2}\left ( \sqrt{2} \right )^{2}}
= \frac{\sqrt{3\cdot 3\cdot 2}-\sqrt{2\cdot 2\cdot 3}}{3-2}
= \frac{3\sqrt{2}-2\sqrt{3}}{1}
= 3\sqrt{2}-2\sqrt{3}
Hence the answer 3\sqrt{2}-2\sqrt{3}
(vii) Answer. 5+2\sqrt{6}
Solution. We have, \frac{\sqrt{3}+\sqrt{2}}{\sqrt{3}-\sqrt{2}}
Rationalising the denominator, we get:
\frac{\sqrt{3}+\sqrt{2}}{\sqrt{3}-\sqrt{2}}\times \frac{\sqrt{3}+\sqrt{2}}{\sqrt{3}+\sqrt{2}}
= \frac{\left ( \sqrt{3} +\sqrt{2}\right )^{2}}{\left ( \sqrt{3} -\sqrt{2} \right )\left ( \sqrt{3} +\sqrt{2} \right )}
Using (a – b) (a + b) = a2 – b2
and (a + b)2 = a2 + b2 + 2ab
= \frac{\left ( \sqrt{3} \right )^{2}+\left ( \sqrt{2} \right )^{2}+2\sqrt{3}\sqrt{2}}{\left ( \sqrt{3} \right )^{2}-\left ( \sqrt{2} \right )^{2}}
= \frac{3+2+2\sqrt{6}}{3-2}
= 5+2\sqrt{6}
Hence the answer is 5+2\sqrt{6}
(viii)
Answer:9+2 \sqrt{15}
Solution:
We have \frac{3 \sqrt{5}+\sqrt{3}}{\sqrt{5}-\sqrt{3}}
Rationalize
=\frac{3 \sqrt{5}+\sqrt{3}}{\sqrt{5}-\sqrt{3}} \times \frac{\sqrt{5}+\sqrt{3}}{\sqrt{5}+\sqrt{3}}=\frac{3 \sqrt{5}(\sqrt{5}+\sqrt{3})+\sqrt{3}(\sqrt{5}+\sqrt{3})}{(\sqrt{5})^{2}-(\sqrt{3})^{2}}
=\frac{15+3 \sqrt{15}+\sqrt{15}+3}{5-3}=\frac{18+4 \sqrt{15}}{2}
=9+2 \sqrt{15}
(ix) Answer:\frac{9+4 \sqrt{6}}{15}
Solution:
We have \frac{4 \sqrt{3}+5 \sqrt{2}}{\sqrt{48}+\sqrt{18}}
=\frac{4 \sqrt{3}+5 \sqrt{2}}{\sqrt{48}+\sqrt{18}}=\frac{4 \sqrt{3}+5 \sqrt{2}}{\sqrt{16 \times 3}+\sqrt{9 \times 2}}=\frac{4 \sqrt{3}+5 \sqrt{2}}{4 \sqrt{3}+3 \sqrt{2}}
Rationalize
=\frac{4 \sqrt{3}+5 \sqrt{2}}{4 \sqrt{3}+3 \sqrt{2}} \times \frac{(4 \sqrt{3}-3 \sqrt{2})}{(4 \sqrt{3}-3 \sqrt{2})}
=\frac{4 \sqrt{3}(4 \sqrt{3}-3 \sqrt{2})+5 \sqrt{2}(4 \sqrt{3}-3 \sqrt{2})}{(4 \sqrt{3})^{2}-(3 \sqrt{2})^{2}}
=\frac{48-12 \sqrt{6}+20 \sqrt{6}-30}{30}
=\frac{18+8 \sqrt{6}}{30}
=\frac{9+4 \sqrt{6}}{15}

Question:11

(i) Find the values of a in each of the following : \frac{5+2\sqrt{3}}{7+4\sqrt{3}}= a-6\sqrt{3}
(ii) Find the values of a in the following : \frac{3-\sqrt{5}}{3+2\sqrt{5}}= a\sqrt{5}-\frac{19}{11}
(iii) Find the values of b in the following : \frac{\sqrt{2}+\sqrt{3}}{3 \sqrt{2}-2 \sqrt{3}}=2-b \sqrt{6}
(iv) Find the values of a and b in the following : \frac{7+\sqrt{5}}{7-\sqrt{5}}-\frac{7-\sqrt{5}}{7+\sqrt{5}}= a+\frac{7}{11}\sqrt{5b}

Answer:

(i) Answer. a = 11
Solution. We have, \frac{5+2\sqrt{3}}{7+4\sqrt{3}}= a-6\sqrt{3}
LHS = \frac{5+2\sqrt{3}}{7+4\sqrt{3}}
Rationalising the denominator, we get:
= \frac{5+2\sqrt{3}}{7+4\sqrt{3}}\times \frac{7-4\sqrt{3}}{7-4\sqrt{3}}
= \frac{\left ( 5+2\sqrt{3} \right )\left ( 7-4\sqrt{3} \right )}{\left ( 7+4\sqrt{3} \right )\left ( 7-4\sqrt{3} \right )}
{Using (a – b) (a + b) = a2 – b2}

= \frac{35+14\sqrt{3}-20\sqrt{3}-24}{7^{2}-\left ( 4\sqrt{3} \right )^{2}}
= \frac{11-6\sqrt{3}}{49-48}
= 11-6\sqrt{3}
Now RHS = a-6\sqrt{3}
\Rightarrow 11-6\sqrt{3}= a-6\sqrt{3}
\Rightarrow 11-6\sqrt{3}= a-6\sqrt{3}
\Rightarrow a= 11
Hence a = 11 is the required answer

(ii)Answer.a= \frac{9}{11}
Solution. Given that, \frac{3-\sqrt{5}}{3+2\sqrt{5}}= a\sqrt{5}-\frac{19}{11}
LHS = \frac{3-\sqrt{5}}{3+2\sqrt{5}}
Rationalising the denominator, we get:
LHS = \frac{3-\sqrt{5}}{3+2\sqrt{5}}\times \frac{3-2\sqrt{5}}{3-2\sqrt{5}}
{Using (a – b) (a + b) = a2 – b2}
= \frac{9-3\sqrt{5}-6\sqrt{5}+10}{3^{2}-\left ( 2\sqrt{5} \right )^{2}}
= \frac{19-9\sqrt{5}}{9-20}
Now RHS = a\sqrt{5}-\frac{19}{11}
\Rightarrow \frac{9\sqrt{5}}{11}-\frac{19}{11}= a\sqrt{5}-\frac{19}{11}
Comparing both , we get
\Rightarrow a= \frac{9}{11}
Hence a= \frac{9}{11}is the correct answer
(iii) Answer:b = -\frac{5 }{6}
Solution:
Given:
\frac{\sqrt{2}+\sqrt{3}}{3 \sqrt{2}-2 \sqrt{3}}=2-b \sqrt{6}
LHS = \frac{\sqrt{2}+\sqrt{3}}{3 \sqrt{2}-2 \sqrt{3}}
Rationalize
= \frac{\sqrt{2}+\sqrt{3}}{3 \sqrt{2}-2 \sqrt{3}} \times \frac{3 \sqrt{2}+2 \sqrt{3}}{3 \sqrt{2}+2 \sqrt{3}}
=\frac{\sqrt{2}(3 \sqrt{2}+2 \sqrt{3})+\sqrt{3}(3 \sqrt{2}+2 \sqrt{3})}{(3 \sqrt{2})^{2}-(2 \sqrt{3})^{2}}
= \frac{6+2 \sqrt{6}+3 \sqrt{6}+6}{18-12}
= 2+\frac{5 \sqrt{6}}{6}
2+\frac{5 \sqrt{6}}{6}=2-b \sqrt{6}
b = -\frac{5 }{6}

(iv) Answer. a = 0, b = 1
Solution. Given,
\frac{7+\sqrt{5}}{7-\sqrt{5}}-\frac{7-\sqrt{5}}{7+\sqrt{5}}= a+\frac{7}{11}\sqrt{5b}
LHS = \frac{7+\sqrt{5}}{7-\sqrt{5}}-\frac{7-\sqrt{5}}{7+\sqrt{5}}
=\frac{\left ( 7+\sqrt{5} \right )\times\left ( 7+\sqrt{5} \right )-\left ( 7-\sqrt{5} \right ) \times \left ( 7-\sqrt{5} \right )}{\left (7 -\sqrt{5} \right )\left (7 +\sqrt{5} \right )}
Using (a – b) (a + b) = a2 – b2
(a + b)2 = a2 + b2 + 2ab
(a - b)2 = a2 + b2 - 2ab
= \frac{\left ( 7^{2} +\sqrt{5}^{2}+2\cdot 7\cdot \sqrt{5}\right )-\left ( 7^{2} +\sqrt{5}^{2}-2\cdot 7\cdot \sqrt{5} \right )}{7^{2}-\sqrt{5}^{2}}
= \frac{\left ( 49+5+14\sqrt{5} \right )-\left ( 49+5-14\sqrt{5} \right )}{49-5}
= \frac{54+14\sqrt{5}-54+14\sqrt{5}}{44}
= \frac{28\sqrt{5}}{44}
RHS = a+\frac{7}{11}\sqrt{5b}
Now LHS = RHS
\Rightarrow \frac{28\sqrt{5}}{44}= a+\frac{7}{11}\sqrt{5b}
\Rightarrow 0+\left ( \frac{4}{4} \right )\frac{7}{11}\sqrt{5}= a+\frac{7}{11}\sqrt{5b}
\Rightarrow a = 0, b = 1
Hence the answer is a = 0, b = 1

Question:12

If a = 2 + \sqrt{3} then find the value of a-\frac{1}{a}

Answer:

Answer. 2\sqrt{3}
Solution. Given that a = 2\sqrt{3}
\therefore We have \frac{1}{a}= \frac{1}{2+\sqrt{3}}
Rationalising,
\Rightarrow \frac{1}{a}= \frac{1}{2+\sqrt{3}}\times \frac{2-\sqrt{3}}{2-\sqrt{3}}
Using (a – b) (a + b) = a2 – b2
\Rightarrow \frac{1}{a}= \frac{2-\sqrt{3}}{2^{2}-\sqrt{3}^{2}}= \frac{2-\sqrt{3}}{4-3}
\Rightarrow \frac{1}{a}=2-\sqrt{3}
Now, a- \frac{1}{a}=2+\sqrt{3}-\left ( 2-\sqrt{3} \right )
\Rightarrow a- \frac{1}{a}=2\sqrt{3}
Hence the answer is 2\sqrt{3}

Question:13

(i) Rationalize the denominator in each of the following and hence evaluate by taking \sqrt{2}= 1\cdot 414,\sqrt{3}= 1\cdot 732 and \sqrt{5}= 2\cdot 236, upto three places of decimal : \frac{4}{\sqrt{3}}
(ii) Rationalize the denominator in each of the following and hence evaluate by taking \sqrt{2}= 1\cdot 414,\sqrt{3}= 1\cdot 732 and \sqrt{5}= 2\cdot 236, upto three places of decimal : \frac{6}{\sqrt{6}}
(iii) Rationalize the denominator in each of the following and hence evaluate by taking \sqrt{2}= 1\cdot 414,\sqrt{3}= 1\cdot 732 and \sqrt{5}= 2\cdot 236, upto three places of decimal : \frac{\sqrt{10}-\sqrt{5}}{\sqrt{2}}
(iv) Rationalize the denominator in each of the following and hence evaluate by taking \sqrt{2}= 1\cdot 414,\sqrt{3}= 1\cdot 732 and \sqrt{5}= 2\cdot 236, upto three places of decimal : \frac{2}{2+\sqrt{2}}
(v) Rationalize the denominator in each of the following and hence evaluate by taking \sqrt{2}= 1\cdot 414,\sqrt{3}= 1\cdot 732 and \sqrt{5}= 2\cdot 236, upto three places of decimal : \frac{1}{\sqrt{3}+\sqrt{2}}

Answer:

(i) Answer. 2.3093
Solution. Given: \frac{4}{\sqrt{3}}
Rationalising,
\frac{4}{\sqrt{3}}\times \frac{\sqrt{3}}{\sqrt{3}}= \frac{4\sqrt{3}}{3}
(Given that \sqrt{3}= 1.732)
= \frac{4\times 1\cdot 732}{3}
= 2.3093
Hence the answer is 2.3093

(ii) Answer. 2.449
Solution. Given: \frac{6}{\sqrt{6}}
Rationalising,
\frac{6}{\sqrt{6}}\times \frac{\sqrt{6}}{\sqrt{6}}= \frac{6\sqrt{6}}{6}
= \frac{6\times \sqrt{2}\sqrt{3}}{6}
Putting the given values,
\sqrt{2}= 1\cdot 414,\sqrt{3}= 1\cdot 732
We get :
= \sqrt{2}\cdot \sqrt{3}
= 1\cdot 414\times 1\cdot 732= 2\cdot 449
Hence the answer is 2.449

(iii) Answer. 0.462852
Solution. Given that \frac{\sqrt{10}-\sqrt{5}}{2}
This can be written as
\frac{\sqrt{2}\times \sqrt{5}-\sqrt{5}}{2}
Now putting the given values,
\sqrt{2}= 1\cdot 414,\sqrt{5}= 2\cdot 236
We get :
\Rightarrow \frac{1\cdot 414\times 2\cdot 236-2\cdot 236}{2}
= 0.462852
Hence the answer is 0.462852
(iv) Answer. 0.414
Solution. Given: \frac{\sqrt{2}}{2+\sqrt{2}}
Rationalising,
\frac{\sqrt{2}}{2+\sqrt{2}}\times \frac{2-\sqrt{2}}{2-\sqrt{2}}
Using (a – b) (a + b) = a2 – b2
= \frac{\sqrt{2}\left ( 2-\sqrt{2} \right )}{2^{2}-\sqrt{2}^{2}}
= \frac{2\sqrt{2}-2}{4-2}
= \frac{2\left ( \sqrt{2}-1 \right )}{2}
= \sqrt{2}-1

Putting the given value of \sqrt{2}= 1\cdot 414
We get
= 1.414 – 1
= 0.414
Hence the answer is 0.414

(v) Answer. 0.318
Solution. Given that \frac{1}{\sqrt{3}+\sqrt{2}}
Rationalising,
\frac{1}{\sqrt{3}+\sqrt{2}}\times \frac{\sqrt{3}-\sqrt{2}}{\sqrt{3}-\sqrt{2}}
Using (a – b) (a + b) = a2 – b2
= \frac{\sqrt{3}-\sqrt{2}}{\sqrt{3}^{2}-\sqrt{2}^{2}}
= \frac{\sqrt{3}-\sqrt{2}}{3-2}
= \sqrt{3}-\sqrt{2}

Putting the given values, \sqrt{2}= 1\cdot 414,\sqrt{3}= 1\cdot 732
We get,
= 1.732 – 1.414
= 0.318
Hence the answer is 0.318

Question:14

(i) Simplify :- (13 + 23 + 33)1/2
(ii)
Simplify :- \left ( \frac{3}{5} \right )^{4}\left ( \frac{8}{5} \right )^{-12}\left ( \frac{32}{5} \right )^{6}
(iii) Simplify :- \left ( \frac{1}{27} \right )^{-\frac{2}{3}}
(iv) Simplify :- \left [ \left \{ \left ( 625 \right )^{-\frac{1}{2}} \right \}^{-\frac{1}{4}} \right ]^{2}
(v) Simplify :- \frac{9^{\frac{1}{3}}\times 27^{\frac{1}{2}}}{3^{\frac{1}{6}}\times 9^{-\frac{2}{3}}}
(vi) Simplify :-64^{-\frac{1}{3}}\left ( 64^{\frac{1}{3}}-64^{\frac{2}{3}} \right )
(vii) Simplify :- \frac{8^{\frac{1}{3}}\times 16^{\frac{1}{3}}}{32^{-\frac{1}{3}}}

Answer:

(i) Answer. 6
Solution. (13 + 23 + 33)1/2
We know that
13 = 1.1.1 = 1
23 = 2.2.2 = 8
33 = 3.3.3 = 27
Putting these values we get
(13 + 23 + 33)1/2= \sqrt{1+8+27}
=\sqrt{36}= 6
Hence the answer is 6

(ii) Answer.\frac{2025}{64}
Solution.\left ( \frac{3}{5} \right )^{4}\left ( \frac{8}{5} \right )^{-12}\left ( \frac{32}{5} \right )^{6}
We know that
8 = 2.2.2 = 23
32 = 2.2.2.2.2 = 25
\left ( \frac{3}{5} \right )^{4}\left ( \frac{8}{5} \right )^{-12}\left ( \frac{32}{5} \right )^{6}= \left ( \frac{3}{5} \right )^{4}\left ( \frac{2^{3}}{5} \right )^{-12}\left ( \frac{2^{5}}{5} \right )^{6}
= \frac{3^{4}\left ( 2^{3} \right )^{-12}\left ( 2^{5} \right )^{6}}{5^{4}5^{-12}5^{6}} \because \left ( \frac{a}{b} \right )^{m}= \frac{a^{m}}{b^{m}}
= \frac{3^{4}2^{-36}2^{30}}{5^{4}5^{-12}5^{6}} \because \left ( \left ( a \right )^{m} \right )^{n}= \left ( a \right )^{mn}
= \frac{3^{4}\times 2^{-36+30}}{5^{4-12+6}} \because \left ( a \right )^{m}\left ( a \right )^{n}= \left ( a \right )^{m+n}
= \frac{3^{4}\times 2^{-6}}{5^{-2}}
= \frac{3^{4}\times 5^{2}}{2^{6}} \because \left ( a \right )^{-m}= \left ( \frac{1}{a} \right )^{m}
= \frac{81\times 25}{64}
= \frac{2025}{64}
Hence the answer is \frac{2025}{64}

(iii) Answer. 9
Solution. Given \left ( \frac{1}{27} \right )^{-\frac{2}{3}}
We know that
27 = 3.3.3 = 33
\left ( \frac{1}{27} \right )^{-\frac{2}{3}}= \left ( \frac{1}{3^{3}} \right )^{-\frac{2}{3}}
= \left ( 3^{3} \right )^{\frac{2}{3}} \because \left ( a \right )^{-m}= \left ( \frac{1}{a} \right )^{m}
= \left ( 3 \right )^{3\times \frac{2}{3}} \because \left ( \left ( a \right )^{m} \right )^{n}= \left ( a \right )^{mn}

= 32 = 9
Hence the answer is 9

(iv) Answer. 5
Solution. Given \left [ \left \{ \left ( 625 \right )^{-\frac{1}{2}} \right \}^{-\frac{1}{4}} \right ]^{2}
We know that
625= \left ( 25 \right )\left ( 25 \right )= 5\cdot 5\cdot 5\cdot 5= 5^{4}
\left [ \left \{ \left ( 625 \right )^{-\frac{1}{2}} \right \}^{-\frac{1}{4}} \right ]^{2}= \left [ \left \{ \left ( \left ( 5 \right )^{4} \right )^{-\frac{1}{2}} \right \} ^{-\frac{1}{4}}\right ]^{2}
= 5^{4\times \frac{-1}{2}\times \frac{-1}{4}\times 2} \because \left ( \left ( a \right )^{m} \right )^{n}= \left ( a \right )^{mn}
= 51 = 5
Hence the answer is 5

(v) Answer.\sqrt[3]{\frac{1}{3}}
Solution. We have \frac{9^{\frac{1}{3}}\times 27^{\frac{1}{2}}}{3^{\frac{1}{6}}\times 9^{-\frac{2}{3}}}
Now we know that
9 = 3.3 = 32
27 = 3.3.3 = 33
\frac{9^{\frac{1}{3}}\times 27^{\frac{1}{2}}}{3^{\frac{1}{6}}\times 9^{-\frac{2}{3}}}= \frac{\left ( 3^{2} \right )^{\frac{1}{3}}\times \left ( 3^{3} \right )^{\frac{1}{2}}}{\left ( 3 \right )^{\frac{1}{6}}\times \left ( 3^{2} \right )^{\tfrac{-2}{3}}}
= \frac{\left ( 3\right )^{2\times \frac{1}{3}}\times \left ( 3 \right )^{3\times \frac{-1}{2}}}{\left ( 3 \right )^{\frac{1}{6}}\times \left ( 3 \right )^{\tfrac{-2}{3}}} \because \left ( \left ( a \right )^{m} \right )^{n}= \left ( a \right )^{mn}
= \frac{\left ( 3 \right )^{\frac{2}{3}-\frac{3}{2}}}{\left ( 3 \right )^{\frac{1}{6}-\frac{2}{3}}} \because \left ( a \right )^{m}\left ( a \right )^{n}= \left ( a \right )^{m+n}
= \frac{3^{\frac{4-9}{6}}}{3^{\frac{1-4}{6}}}
=\frac{3^{-\frac{5}{6}}}{3^{-\frac{3}{6}}}
= 3^{\frac{-5}{6}-\left ( \frac{-3}{6} \right )} \because \frac{\left ( a \right )^{m}}{\left ( a \right )^{n}}= \left ( a \right )^{m-n}
= 3^{-\frac{2}{6}}
= \left ( \frac{1}{3} \right )^{\frac{1}{3}} \because \left ( a \right )^{-m}= \left ( \frac{1}{a} \right )^{m}
= \sqrt[3]{\frac{1}{3}}

Hence the answer is \sqrt[3]{\frac{1}{3}}

(vi) Answer. – 3
Solution. We have ,64^{-\frac{1}{3}}\left ( 64^{\frac{1}{3}}-64^{\frac{2}{3}} \right )
We know that 64 =4.4.4=43
= \left ( 4^{3} \right )^{\frac{-1}{3}}\left \{ \left ( \left ( 4^{3} \right ) ^{\frac{1}{3}}-\left ( 4^{3} \right ) ^{\frac{2}{3}}\right )\right \}
= \left ( 4 \right )^{3\times \frac{-1}{3}}\left \{ \left ( \left ( 4 \right )^{3\times \frac{1}{3}}-\left ( 4 \right )^{3\times \frac{2}{3}} \right ) \right \} \because \left ( \left ( a \right )^{m} \right )^{n}= \left ( a \right )^{mn}
= 4^{-1}\left ( 4-4^{2} \right )
= \frac{1}{4}\left ( 4-16 \right ) \because \left ( a \right )^{-m}= \left ( \frac{1}{a} \right )^{m}
= \frac{1}{4}\left ( -12 \right )

= – 3
Hence the answer is – 3

(vii) Answer. 16
Solution.
Given ,\frac{8^{\frac{1}{3}}\times 16^{\frac{1}{3}}}{32^{-\frac{1}{3}}}
We know that
8 = 2.2.2 = 23
16 = 2.2.2.2 = 24
32 = 2.2.2.2.2 = 25
\frac{8^{\frac{1}{3}}\times 16^{\frac{1}{3}}}{32^{-\frac{1}{3}}}= \frac{\left ( 2^{3} \right )^{\frac{1}{3}}\times\left ( 2^{4} \right )^{\frac{1}{3}}}{\left ( 2^{5} \right )^{-\frac{1}{3}}}
= \frac{2^{3\times\frac{1}{3}}\times2^{4\times\frac{1}{3}}}{2^{5\times\frac{-1}{3}}} \because \left ( \left ( a \right )^{m} \right )^{n}= \left ( a \right )^{mn}
= 2^{1+\frac{4}{3}+\frac{5}{3}} \because \left ( a \right )^{m}\left ( a \right )^{n}= \left ( a \right )^{m+n} and
\because \frac{\left ( a \right )^{m}}{\left ( a \right )^{n}}= \left ( a \right )^{m-n}
= 2^{\frac{3+4+5}{3}}= 2^{\frac{12}{3}}
= 2^{4}= 16

Hence the answer is 16.

NCERT Exemplar Class 9 Maths Solutions Chapter-1: Exercise-1.4

Question:1

Express 0.6 + 0\cdot \bar{7}+0\cdot \overline{47} in the form \frac{p}{q} where p and q are integers and q\neq 0.
Answer:

Answer. \frac{167}{90}
Solution. Let x = 0.6
Multiply by 10 on LHS and RHS
10x = 6
x= \frac{6}{10}
x= \frac{3}{5}
So, the \frac{p}{q} from of 0.6 = \frac{3}{5}
Let y = 0\cdot \bar{7}
Multiply by 10 on LHS and RHS
10y = 7.7777 …….
10y – y = 7\cdot \bar{7}-0\cdot \bar{7}
= 7.77777 ….. – 0.77777 ……
9y = 7
y= \frac{7}{9}
So the \frac{p}{q} from of 0.7777 = \frac{7}{9}
Let z = 0.47777…
Multiply by 10 on both side
10z = 4.7777 ….
10z – z = 4\cdot \bar{7}-0\cdot4\bar{7}
9z = 4.3
z\approx \frac{4\cdot 3}{9}
z= \frac{43}{90}
So the \frac{p}{q}from of 0.4777 …. = \frac{43}{90}
Therefore, \frac{p}{q} form of 0.6 + 0\cdot \bar{7}+0\cdot4\bar{7} is,
x+y+z= \frac{3}{5}+\frac{7}{9}+\frac{43}{90}
= \frac{\left ( 54+70+43 \right )}{90}
= \frac{167}{90}
Hence the answer is \frac{167}{90}

Question:2

Simplify :- \frac{7\sqrt{3}}{\sqrt{10}+\sqrt{3}}-\frac{2\sqrt{5}}{\sqrt{6}+\sqrt{5}}-\frac{3\sqrt{2}}{\sqrt{15}+3\sqrt{2}}

Answer:

Answer. 1
Solution.
\frac{7\sqrt{3}}{\sqrt{10}+\sqrt{3}}-\frac{2\sqrt{5}}{\sqrt{6}+\sqrt{5}}-\frac{3\sqrt{2}}{\sqrt{15}+3\sqrt{2}}
Rationalise the denominators:
\Rightarrow \left ( \frac{7\sqrt{3}}{\sqrt{10}+\sqrt{3}}\times \frac{\sqrt{10}-\sqrt{3}}{\sqrt{10}-\sqrt{3}} \right )-\left ( \frac{2\sqrt{5}}{\sqrt{6}+\sqrt{3}} \times \frac{\sqrt{6}-\sqrt{5}}{\sqrt{6}-\sqrt{5}}\right )-\left ( \frac{3\sqrt{2}}{\sqrt{15}+3\sqrt{2} } \times\frac{\sqrt{15}-3\sqrt{2}}{\sqrt{15}-3\sqrt{2}}\right )
\Rightarrow \frac{7\sqrt{3}\left ( \sqrt{10} -\sqrt{3}\right )}{10-3}-\frac{2\sqrt{5}\left ( \sqrt{6}-\sqrt{5} \right )}{6-5}-\frac{3\sqrt{2}\left ( \sqrt{15}-3\sqrt{2} \right )}{15-8}
\left [ \because a^{2}-b^{2} = \left ( a+b \right )\left ( a-b \right )\right ]
\Rightarrow \frac{7\sqrt{3}\left ( \sqrt{10} -\sqrt{3}\right )}{7}-\frac{2\sqrt{5}\left ( \sqrt{6}-\sqrt{5} \right )}{1}-\frac{3\sqrt{2}\left ( \sqrt{15}-3\sqrt{2} \right )}{3}
\Rightarrow \frac{7\sqrt{30}-21}{7}-\frac{2\sqrt{30}-10}{1}+\frac{3\sqrt{30}-18}{3}
\Rightarrow \frac{21\sqrt{30}-63-42\sqrt{30}+210+21\sqrt{30}-126}{21}
\Rightarrow \frac{21}{21}= 1
Hence the answer is 1.

Question:3

If \sqrt{2}= 1\cdot 414 and \sqrt{3}= 1\cdot 732 then find the value of \frac{4}{3\sqrt{3}-2\sqrt{2}}+\frac{3}{3\sqrt{3}+2\sqrt{2}}

Answer:

Answer. 2.0632
Solution. Given that :
\sqrt{2}= 1\cdot 414,\sqrt{3}= 1\cdot 732
\frac{4}{3\sqrt{3}-2\sqrt{2}}+\frac{3}{3\sqrt{3}+2\sqrt{2}}
= \frac{4\left ( 3\sqrt{3}+2\sqrt{2} \right )}{\left ( 3\sqrt{3}-2\sqrt{2} \right )\left ( 3\sqrt{3}+2\sqrt{2} \right )}+\frac{3\left ( 3\sqrt{3}-2\sqrt{2} \right )}{\left ( 3\sqrt{3}+2\sqrt{2} \right )\left (3\sqrt{3}-2\sqrt{2} \right )}
= \frac{4\left ( 3\sqrt{3}+2\sqrt{2} \right )+3\left ( 3\sqrt{3}-2\sqrt{2} \right )}{\left ( 3\sqrt{3}-2\sqrt{2} \right )\left ( 3\sqrt{3}+2\sqrt{2} \right )}
Using (a – b) (a + b) = a2 – b2
= \frac{12\sqrt{3}+8\sqrt{2}+9\sqrt{3}-6\sqrt{2}}{\left ( 3\sqrt{3} \right )^{2}-\left ( 2\sqrt{2} \right )^{2}}
= \frac{21\sqrt{3}+2\sqrt{2}}{27-8}
= \frac{21\sqrt{3}+2\sqrt{2}}{19}

Putting the given values,= \frac{21\left ( 1\cdot 732 \right )+2\left ( 1\cdot 414 \right )}{19}
=\frac{39\cdot 2014}{19}
= 2.0632
Hence the answer is 2.0632.

Question:4

If a= \frac{3+\sqrt{5}}{2} then find the value of a^{2}= \frac{1}{a^{2}}

Answer:

Answer. 7
Solution.
Given that :- a= \frac{3+\sqrt{5}}{2}
\therefore \frac{1}{a}= \frac{2}{3+\sqrt{5}}
On rationalizing the denominator, we get
\frac{1}{a}= \frac{2\left ( 3-\sqrt{5} \right )}{\left ( 3+\sqrt{5} \right )\left ( 3-\sqrt{5} \right )}
Using (a – b) (a + b) = a2 – b2
= \frac{6-2\sqrt{5}}{3^{2}-\sqrt{5}^{2}}
= \frac{6-2\sqrt{5}}{9-5}
= \frac{6-2\sqrt{5}}{4}
= \frac{3-\sqrt{5}}{2}
Also, \left ( a+\frac{1}{a} \right )^{2}= a^{2}+\frac{1}{a^{2}}+2
Substituting the values of a and \frac{1}{a}
We get, \left ( \frac{3+\sqrt{5}}{2}+\frac{3-\sqrt{5}}{2} \right )^{2}= \left ( a^{2}+\frac{1}{a^{2}}+2 \right )
\therefore a^{2}+\frac{1}{a^{2}}+2= \left ( \frac{3+\sqrt{5}+3-\sqrt{5}}{2} \right )^{2}
= (3)2 = 9
\therefore a^{2}+\frac{1}{a^{2}}= 9-2= 7
Hence the correct answer is 7.

Question:5

If x= \frac{\sqrt{3}+\sqrt{2}}{\sqrt{3}-\sqrt{2}} and y= \frac{\sqrt{3}-\sqrt{2}}{\sqrt{3}+\sqrt{2}} then find the value of x2 + y2.

Answer:

Answer. 98
Solution.We use the identity \left ( a+b \right )^{2}= a^{2}+b^{2}+2ab
So, \left ( \sqrt{a}+\sqrt{b} \right )^{2}= a+2\sqrt{ab}+b
x^{2}+y^{2}= \left ( \frac{\sqrt{3}+\sqrt{2}}{\sqrt{3}-\sqrt{2}} \right )^{2}+\left ( \frac{\sqrt{3}-\sqrt{2}}{\sqrt{3}+\sqrt{2}} \right )^{2}
= \frac{3+2\sqrt{6}+2}{3-2\sqrt{6}+2}+\frac{3-2\sqrt{6}+2}{3+2\sqrt{6}+2}
= \frac{5+2\sqrt{6}}{5-2\sqrt{6}}+\frac{5-2\sqrt{6}}{5+2\sqrt{6}}
= \frac{\left ( 5+2\sqrt{6} \right )^{2}+\left ( 5-2\sqrt{6} \right )^{2}}{\left ( 5-2\sqrt{6} \right ){\left ( 5+2\sqrt{6} \right )}}
= \frac{\left ( 25+20\sqrt{6}+24 \right )+\left ( 25-20\sqrt{6} +24\right )}{25-24}

Using (a – b) (a + b) = a2– b2= 98
Hence the answer is 98.

Question:6

Simplify : \left ( 256 \right )^{-\left ( 4^{\frac{-3}{2}} \right )}

Answer:

Answer. 248
Solution. Given, \left ( 256 \right )^{-\left ( 4^{\frac{-3}{2}} \right )}
We know that,
256 = 2.2.2.2.2.2.2.2 = 28
\left ( 256 \right )^{-\left ( 4^{\frac{-3}{2}} \right )} = \left ( 2^{8} \right )^{\left ( -4 \right )\times \left ( -\frac{3}{2} \right )}
\because \left ( \left ( a \right ) ^{m}\right )^{n}= \left ( a \right )^{mn}
=\left ( 2 \right )^{8\times \left ( -4 \right )\times \left ( -\frac{3}{2} \right )}
= 2^{8\times 4\times \frac{3}{2}}= 2^{8\times 2\times 3}= 2^{48}
Hence the answer is 248

Question:7

Find the value of \frac{4}{\left ( 216 \right )^{\frac{-2}{3}}}+\frac{1}{\left ( 256 \right )^{\frac{-3}{4}}}+\frac{2}{\left ( 243 \right )^{\frac{-1}{5}}}.

Answer:

Answer. 214
Solution. We have, \frac{4}{\left ( 216 \right )^{\frac{-2}{3}}}+\frac{1}{\left ( 256 \right )^{\frac{-3}{4}}}+\frac{2}{\left ( 243 \right )^{\frac{-1}{5}}}
We know that
216 = 6.6.6 = 63
256 = 4.4.4.4 = 44
243 = 3.3.3.3.3 = 35
So, \frac{4}{\left ( 216 \right )^{\frac{-2}{3}}}+\frac{1}{\left ( 256 \right )^{\frac{-3}{4}}}+\frac{2}{\left ( 243 \right )^{\frac{-1}{5}}}
= \frac{4}{\left ( 6^{3} \right )^{\frac{-2}{3}}}+\frac{1}{\left ( 4^{4} \right )^{\frac{-3}{4}}}+\frac{2}{\left ( 3 ^{5}\right )^{-\frac{1}{5}}}
= \frac{4}{\left ( 6 \right )^{3\times \frac{-2}{3}}}+\frac{1}{\left ( 4 \right )^{4\times \frac{-3}{4}}}+\frac{2}{\left ( 3 \right )^{5\times \frac{-1}{5}}}
\because \left ( \left ( a \right )^{m} \right )^{n}= \left ( a \right )^{mn}
= \frac{4}{6^{-2}}+\frac{1}{4^{-3}}+\frac{2}{3^{-1}}
= 4 × 62 + 43 + 2 × 3
\because \frac{1}{\left ( a \right )^{-n}}= \left ( a \right )^{n}
= 4 × 36 + 64 + 6
= 144 + 70
= 214
Hence the answer is 214

NCERT Exemplar Solutions Class 9 Maths Chapter 1 Number System Important Topics:

Topics covered in NCERT exemplar Class 9 Maths solutions chapter 1 deals with the understanding of:

◊ Real Numbers as Rational numbers and Irrational numbers.

◊ Problems to find a rational number between two given numbers or to find an irrational number between two given numbers are dealt with here.

◊ The concepts of the number line and locating rational number and irrational numbers on the number line are explained.

◊ The decimal expressions of real numbers and fractions are explained in categories like terminating or non-terminating as well as recurring or non-recurring.

◊ Questions based on operations performed on real numbers and exponents for real numbers are explained appropriately.

◊ Laws of exponents for real numbers such as dealing with multiplications of powers and addition of powers of real numbers are explained through NCERT exemplar Class 9 Maths solutions chapter 1.

◊ The concept of rationalising the denominator by multiplication of conjugate of an irrational number is explained through NCERT exemplar solutions for Class 9 Maths chapter 1.

NCERT Class 9 Maths Exemplar Solutions for Other Chapters:

Features of NCERT Exemplar Class 9 Maths Solutions Chapter 1 Number System:

These Class 9 Maths NCERT exemplar solutions chapter 1 will help the students to understand the concept and knowledge of the real numbers. Students of Class 9 can use these solutions as reference material for better study and practice sums of real numbers.

The NCERT exemplar Class 9 Maths solutions chapter 1 Number System will be adequate to solve problems of other reference books such as NCERT Class 9 Maths, RD Sharma Class 9 Maths or RS Aggarwal Class 9 Maths etcetera.

NCERT exemplar Class 9 Maths solutions chapter 1 pdf download will be made available to resolve the doubts encountered while attempting the exemplar of chapter 1.

NCERT Class 9 Exemplar Solutions for Other Subjects:

Check NCERT Solutions for questions given in book

Also, check NCERT Solution Subject Wise

Also, check NCERT Notes Subject Wise

Also Check NCERT Books and NCERT Syllabus here

Frequently Asked Question (FAQs)

1. What is the weightage of Number System in Class 9 Maths?

Number system weighs approximately 8-10% (discretional to the paper setters; varies from school to school for Class 9 Maths)  of the total marks of the paper, however, being one on the basic building blocks for Maths of higher Classes (Class 10, Class 11 and Class 12), the student should understand and practice NCERT exemplar Class 9 Maths chapter 1.

2. Will these problems of exemplar require additional knowledge of real number?

The NCERT exemplar Class 9 Maths solutions chapter 1 equip the student with a multidimensional approach to the problems and understanding the concept of Number System.

3. What is the weightage of Real Numbers in JEE Main and JEE Advanced?

A clear understanding of Real Number can prepare a student in solving the problems based on fundamentals of maths which ranges from 2-5% marks of the whole paper.

4. Will it be beneficial to refer NCERT exemplar Class 9 Maths solutions for competitive exams?

NCERT exemplar Class 9 Maths solutions chapter 1 will provide the student multifaceted problems to hone the skills and prepare well for the competitive exams such as JEE Main.

5. Why π is an irrational number even its value is 22/7?

Pi is an irrational number because it is a non-terminating non-recurring decimal number. Recently, a tech company employee has broken the Guinness World Record by calculating Pi till 31.4 trillion (1012) decimal places (See it is never-ending…..)

6. Is the number of rational numbers more than the number of irrational numbers?

NCERT exemplar Class 9 Maths solutions chapter 1 explains that both the sets of rational and irrational numbers are infinite; hence, it cannot be deduced.

Articles

Get answers from students and experts

A block of mass 0.50 kg is moving with a speed of 2.00 ms-1 on a smooth surface. It strikes another mass of 1.00 kg and then they move together as a single body. The energy loss during the collision is

Option 1)

0.34\; J

Option 2)

0.16\; J

Option 3)

1.00\; J

Option 4)

0.67\; J

A person trying to lose weight by burning fat lifts a mass of 10 kg upto a height of 1 m 1000 times.  Assume that the potential energy lost each time he lowers the mass is dissipated.  How much fat will he use up considering the work done only when the weight is lifted up ?  Fat supplies 3.8×107 J of energy per kg which is converted to mechanical energy with a 20% efficiency rate.  Take g = 9.8 ms−2 :

Option 1)

2.45×10−3 kg

Option 2)

 6.45×10−3 kg

Option 3)

 9.89×10−3 kg

Option 4)

12.89×10−3 kg

 

An athlete in the olympic games covers a distance of 100 m in 10 s. His kinetic energy can be estimated to be in the range

Option 1)

2,000 \; J - 5,000\; J

Option 2)

200 \, \, J - 500 \, \, J

Option 3)

2\times 10^{5}J-3\times 10^{5}J

Option 4)

20,000 \, \, J - 50,000 \, \, J

A particle is projected at 600   to the horizontal with a kinetic energy K. The kinetic energy at the highest point

Option 1)

K/2\,

Option 2)

\; K\;

Option 3)

zero\;

Option 4)

K/4

In the reaction,

2Al_{(s)}+6HCL_{(aq)}\rightarrow 2Al^{3+}\, _{(aq)}+6Cl^{-}\, _{(aq)}+3H_{2(g)}

Option 1)

11.2\, L\, H_{2(g)}  at STP  is produced for every mole HCL_{(aq)}  consumed

Option 2)

6L\, HCl_{(aq)}  is consumed for ever 3L\, H_{2(g)}      produced

Option 3)

33.6 L\, H_{2(g)} is produced regardless of temperature and pressure for every mole Al that reacts

Option 4)

67.2\, L\, H_{2(g)} at STP is produced for every mole Al that reacts .

How many moles of magnesium phosphate, Mg_{3}(PO_{4})_{2} will contain 0.25 mole of oxygen atoms?

Option 1)

0.02

Option 2)

3.125 × 10-2

Option 3)

1.25 × 10-2

Option 4)

2.5 × 10-2

If we consider that 1/6, in place of 1/12, mass of carbon atom is taken to be the relative atomic mass unit, the mass of one mole of a substance will

Option 1)

decrease twice

Option 2)

increase two fold

Option 3)

remain unchanged

Option 4)

be a function of the molecular mass of the substance.

With increase of temperature, which of these changes?

Option 1)

Molality

Option 2)

Weight fraction of solute

Option 3)

Fraction of solute present in water

Option 4)

Mole fraction.

Number of atoms in 558.5 gram Fe (at. wt.of Fe = 55.85 g mol-1) is

Option 1)

twice that in 60 g carbon

Option 2)

6.023 × 1022

Option 3)

half that in 8 g He

Option 4)

558.5 × 6.023 × 1023

A pulley of radius 2 m is rotated about its axis by a force F = (20t - 5t2) newton (where t is measured in seconds) applied tangentially. If the moment of inertia of the pulley about its axis of rotation is 10 kg m2 , the number of rotations made by the pulley before its direction of motion if reversed, is

Option 1)

less than 3

Option 2)

more than 3 but less than 6

Option 3)

more than 6 but less than 9

Option 4)

more than 9

Data Administrator

Database professionals use software to store and organise data such as financial information, and customer shipping records. Individuals who opt for a career as data administrators ensure that data is available for users and secured from unauthorised sales. DB administrators may work in various types of industries. It may involve computer systems design, service firms, insurance companies, banks and hospitals.

4 Jobs Available
Bio Medical Engineer

The field of biomedical engineering opens up a universe of expert chances. An Individual in the biomedical engineering career path work in the field of engineering as well as medicine, in order to find out solutions to common problems of the two fields. The biomedical engineering job opportunities are to collaborate with doctors and researchers to develop medical systems, equipment, or devices that can solve clinical problems. Here we will be discussing jobs after biomedical engineering, how to get a job in biomedical engineering, biomedical engineering scope, and salary. 

4 Jobs Available
Ethical Hacker

A career as ethical hacker involves various challenges and provides lucrative opportunities in the digital era where every giant business and startup owns its cyberspace on the world wide web. Individuals in the ethical hacker career path try to find the vulnerabilities in the cyber system to get its authority. If he or she succeeds in it then he or she gets its illegal authority. Individuals in the ethical hacker career path then steal information or delete the file that could affect the business, functioning, or services of the organization.

3 Jobs Available
GIS Expert

GIS officer work on various GIS software to conduct a study and gather spatial and non-spatial information. GIS experts update the GIS data and maintain it. The databases include aerial or satellite imagery, latitudinal and longitudinal coordinates, and manually digitized images of maps. In a career as GIS expert, one is responsible for creating online and mobile maps.

3 Jobs Available
Data Analyst

The invention of the database has given fresh breath to the people involved in the data analytics career path. Analysis refers to splitting up a whole into its individual components for individual analysis. Data analysis is a method through which raw data are processed and transformed into information that would be beneficial for user strategic thinking.

Data are collected and examined to respond to questions, evaluate hypotheses or contradict theories. It is a tool for analyzing, transforming, modeling, and arranging data with useful knowledge, to assist in decision-making and methods, encompassing various strategies, and is used in different fields of business, research, and social science.

3 Jobs Available
Geothermal Engineer

Individuals who opt for a career as geothermal engineers are the professionals involved in the processing of geothermal energy. The responsibilities of geothermal engineers may vary depending on the workplace location. Those who work in fields design facilities to process and distribute geothermal energy. They oversee the functioning of machinery used in the field.

3 Jobs Available
Database Architect

If you are intrigued by the programming world and are interested in developing communications networks then a career as database architect may be a good option for you. Data architect roles and responsibilities include building design models for data communication networks. Wide Area Networks (WANs), local area networks (LANs), and intranets are included in the database networks. It is expected that database architects will have in-depth knowledge of a company's business to develop a network to fulfil the requirements of the organisation. Stay tuned as we look at the larger picture and give you more information on what is db architecture, why you should pursue database architecture, what to expect from such a degree and what your job opportunities will be after graduation. Here, we will be discussing how to become a data architect. Students can visit NIT Trichy, IIT Kharagpur, JMI New Delhi

3 Jobs Available
Remote Sensing Technician

Individuals who opt for a career as a remote sensing technician possess unique personalities. Remote sensing analysts seem to be rational human beings, they are strong, independent, persistent, sincere, realistic and resourceful. Some of them are analytical as well, which means they are intelligent, introspective and inquisitive. 

Remote sensing scientists use remote sensing technology to support scientists in fields such as community planning, flight planning or the management of natural resources. Analysing data collected from aircraft, satellites or ground-based platforms using statistical analysis software, image analysis software or Geographic Information Systems (GIS) is a significant part of their work. Do you want to learn how to become remote sensing technician? There's no need to be concerned; we've devised a simple remote sensing technician career path for you. Scroll through the pages and read.

3 Jobs Available
Budget Analyst

Budget analysis, in a nutshell, entails thoroughly analyzing the details of a financial budget. The budget analysis aims to better understand and manage revenue. Budget analysts assist in the achievement of financial targets, the preservation of profitability, and the pursuit of long-term growth for a business. Budget analysts generally have a bachelor's degree in accounting, finance, economics, or a closely related field. Knowledge of Financial Management is of prime importance in this career.

4 Jobs Available
Data Analyst

The invention of the database has given fresh breath to the people involved in the data analytics career path. Analysis refers to splitting up a whole into its individual components for individual analysis. Data analysis is a method through which raw data are processed and transformed into information that would be beneficial for user strategic thinking.

Data are collected and examined to respond to questions, evaluate hypotheses or contradict theories. It is a tool for analyzing, transforming, modeling, and arranging data with useful knowledge, to assist in decision-making and methods, encompassing various strategies, and is used in different fields of business, research, and social science.

3 Jobs Available
Underwriter

An underwriter is a person who assesses and evaluates the risk of insurance in his or her field like mortgage, loan, health policy, investment, and so on and so forth. The underwriter career path does involve risks as analysing the risks means finding out if there is a way for the insurance underwriter jobs to recover the money from its clients. If the risk turns out to be too much for the company then in the future it is an underwriter who will be held accountable for it. Therefore, one must carry out his or her job with a lot of attention and diligence.

3 Jobs Available
Finance Executive
3 Jobs Available
Product Manager

A Product Manager is a professional responsible for product planning and marketing. He or she manages the product throughout the Product Life Cycle, gathering and prioritising the product. A product manager job description includes defining the product vision and working closely with team members of other departments to deliver winning products.  

3 Jobs Available
Operations Manager

Individuals in the operations manager jobs are responsible for ensuring the efficiency of each department to acquire its optimal goal. They plan the use of resources and distribution of materials. The operations manager's job description includes managing budgets, negotiating contracts, and performing administrative tasks.

3 Jobs Available
Stock Analyst

Individuals who opt for a career as a stock analyst examine the company's investments makes decisions and keep track of financial securities. The nature of such investments will differ from one business to the next. Individuals in the stock analyst career use data mining to forecast a company's profits and revenues, advise clients on whether to buy or sell, participate in seminars, and discussing financial matters with executives and evaluate annual reports.

2 Jobs Available
Researcher

A Researcher is a professional who is responsible for collecting data and information by reviewing the literature and conducting experiments and surveys. He or she uses various methodological processes to provide accurate data and information that is utilised by academicians and other industry professionals. Here, we will discuss what is a researcher, the researcher's salary, types of researchers.

2 Jobs Available
Welding Engineer

Welding Engineer Job Description: A Welding Engineer work involves managing welding projects and supervising welding teams. He or she is responsible for reviewing welding procedures, processes and documentation. A career as Welding Engineer involves conducting failure analyses and causes on welding issues. 

5 Jobs Available
Transportation Planner

A career as Transportation Planner requires technical application of science and technology in engineering, particularly the concepts, equipment and technologies involved in the production of products and services. In fields like land use, infrastructure review, ecological standards and street design, he or she considers issues of health, environment and performance. A Transportation Planner assigns resources for implementing and designing programmes. He or she is responsible for assessing needs, preparing plans and forecasts and compliance with regulations.

3 Jobs Available
Environmental Engineer

Individuals who opt for a career as an environmental engineer are construction professionals who utilise the skills and knowledge of biology, soil science, chemistry and the concept of engineering to design and develop projects that serve as solutions to various environmental problems. 

2 Jobs Available
Safety Manager

A Safety Manager is a professional responsible for employee’s safety at work. He or she plans, implements and oversees the company’s employee safety. A Safety Manager ensures compliance and adherence to Occupational Health and Safety (OHS) guidelines.

2 Jobs Available
Conservation Architect

A Conservation Architect is a professional responsible for conserving and restoring buildings or monuments having a historic value. He or she applies techniques to document and stabilise the object’s state without any further damage. A Conservation Architect restores the monuments and heritage buildings to bring them back to their original state.

2 Jobs Available
Structural Engineer

A Structural Engineer designs buildings, bridges, and other related structures. He or she analyzes the structures and makes sure the structures are strong enough to be used by the people. A career as a Structural Engineer requires working in the construction process. It comes under the civil engineering discipline. A Structure Engineer creates structural models with the help of computer-aided design software. 

2 Jobs Available
Highway Engineer

Highway Engineer Job Description: A Highway Engineer is a civil engineer who specialises in planning and building thousands of miles of roads that support connectivity and allow transportation across the country. He or she ensures that traffic management schemes are effectively planned concerning economic sustainability and successful implementation.

2 Jobs Available
Field Surveyor

Are you searching for a Field Surveyor Job Description? A Field Surveyor is a professional responsible for conducting field surveys for various places or geographical conditions. He or she collects the required data and information as per the instructions given by senior officials. 

2 Jobs Available
Orthotist and Prosthetist

Orthotists and Prosthetists are professionals who provide aid to patients with disabilities. They fix them to artificial limbs (prosthetics) and help them to regain stability. There are times when people lose their limbs in an accident. In some other occasions, they are born without a limb or orthopaedic impairment. Orthotists and prosthetists play a crucial role in their lives with fixing them to assistive devices and provide mobility.

6 Jobs Available
Pathologist

A career in pathology in India is filled with several responsibilities as it is a medical branch and affects human lives. The demand for pathologists has been increasing over the past few years as people are getting more aware of different diseases. Not only that, but an increase in population and lifestyle changes have also contributed to the increase in a pathologist’s demand. The pathology careers provide an extremely huge number of opportunities and if you want to be a part of the medical field you can consider being a pathologist. If you want to know more about a career in pathology in India then continue reading this article.

5 Jobs Available
Veterinary Doctor
5 Jobs Available
Speech Therapist
4 Jobs Available
Gynaecologist

Gynaecology can be defined as the study of the female body. The job outlook for gynaecology is excellent since there is evergreen demand for one because of their responsibility of dealing with not only women’s health but also fertility and pregnancy issues. Although most women prefer to have a women obstetrician gynaecologist as their doctor, men also explore a career as a gynaecologist and there are ample amounts of male doctors in the field who are gynaecologists and aid women during delivery and childbirth. 

4 Jobs Available
Audiologist

The audiologist career involves audiology professionals who are responsible to treat hearing loss and proactively preventing the relevant damage. Individuals who opt for a career as an audiologist use various testing strategies with the aim to determine if someone has a normal sensitivity to sounds or not. After the identification of hearing loss, a hearing doctor is required to determine which sections of the hearing are affected, to what extent they are affected, and where the wound causing the hearing loss is found. As soon as the hearing loss is identified, the patients are provided with recommendations for interventions and rehabilitation such as hearing aids, cochlear implants, and appropriate medical referrals. While audiology is a branch of science that studies and researches hearing, balance, and related disorders.

3 Jobs Available
Oncologist

An oncologist is a specialised doctor responsible for providing medical care to patients diagnosed with cancer. He or she uses several therapies to control the cancer and its effect on the human body such as chemotherapy, immunotherapy, radiation therapy and biopsy. An oncologist designs a treatment plan based on a pathology report after diagnosing the type of cancer and where it is spreading inside the body.

3 Jobs Available
Anatomist

Are you searching for an ‘Anatomist job description’? An Anatomist is a research professional who applies the laws of biological science to determine the ability of bodies of various living organisms including animals and humans to regenerate the damaged or destroyed organs. If you want to know what does an anatomist do, then read the entire article, where we will answer all your questions.

2 Jobs Available
Actor

For an individual who opts for a career as an actor, the primary responsibility is to completely speak to the character he or she is playing and to persuade the crowd that the character is genuine by connecting with them and bringing them into the story. This applies to significant roles and littler parts, as all roles join to make an effective creation. Here in this article, we will discuss how to become an actor in India, actor exams, actor salary in India, and actor jobs. 

4 Jobs Available
Acrobat

Individuals who opt for a career as acrobats create and direct original routines for themselves, in addition to developing interpretations of existing routines. The work of circus acrobats can be seen in a variety of performance settings, including circus, reality shows, sports events like the Olympics, movies and commercials. Individuals who opt for a career as acrobats must be prepared to face rejections and intermittent periods of work. The creativity of acrobats may extend to other aspects of the performance. For example, acrobats in the circus may work with gym trainers, celebrities or collaborate with other professionals to enhance such performance elements as costume and or maybe at the teaching end of the career.

3 Jobs Available
Video Game Designer

Career as a video game designer is filled with excitement as well as responsibilities. A video game designer is someone who is involved in the process of creating a game from day one. He or she is responsible for fulfilling duties like designing the character of the game, the several levels involved, plot, art and similar other elements. Individuals who opt for a career as a video game designer may also write the codes for the game using different programming languages.

Depending on the video game designer job description and experience they may also have to lead a team and do the early testing of the game in order to suggest changes and find loopholes.

3 Jobs Available
Radio Jockey

Radio Jockey is an exciting, promising career and a great challenge for music lovers. If you are really interested in a career as radio jockey, then it is very important for an RJ to have an automatic, fun, and friendly personality. If you want to get a job done in this field, a strong command of the language and a good voice are always good things. Apart from this, in order to be a good radio jockey, you will also listen to good radio jockeys so that you can understand their style and later make your own by practicing.

A career as radio jockey has a lot to offer to deserving candidates. If you want to know more about a career as radio jockey, and how to become a radio jockey then continue reading the article.

3 Jobs Available
Choreographer

The word “choreography" actually comes from Greek words that mean “dance writing." Individuals who opt for a career as a choreographer create and direct original dances, in addition to developing interpretations of existing dances. A Choreographer dances and utilises his or her creativity in other aspects of dance performance. For example, he or she may work with the music director to select music or collaborate with other famous choreographers to enhance such performance elements as lighting, costume and set design.

2 Jobs Available
Social Media Manager

A career as social media manager involves implementing the company’s or brand’s marketing plan across all social media channels. Social media managers help in building or improving a brand’s or a company’s website traffic, build brand awareness, create and implement marketing and brand strategy. Social media managers are key to important social communication as well.

2 Jobs Available
Photographer

Photography is considered both a science and an art, an artistic means of expression in which the camera replaces the pen. In a career as a photographer, an individual is hired to capture the moments of public and private events, such as press conferences or weddings, or may also work inside a studio, where people go to get their picture clicked. Photography is divided into many streams each generating numerous career opportunities in photography. With the boom in advertising, media, and the fashion industry, photography has emerged as a lucrative and thrilling career option for many Indian youths.

2 Jobs Available
Producer

An individual who is pursuing a career as a producer is responsible for managing the business aspects of production. They are involved in each aspect of production from its inception to deception. Famous movie producers review the script, recommend changes and visualise the story. 

They are responsible for overseeing the finance involved in the project and distributing the film for broadcasting on various platforms. A career as a producer is quite fulfilling as well as exhaustive in terms of playing different roles in order for a production to be successful. Famous movie producers are responsible for hiring creative and technical personnel on contract basis.

2 Jobs Available
Copy Writer

In a career as a copywriter, one has to consult with the client and understand the brief well. A career as a copywriter has a lot to offer to deserving candidates. Several new mediums of advertising are opening therefore making it a lucrative career choice. Students can pursue various copywriter courses such as Journalism, Advertising, Marketing Management. Here, we have discussed how to become a freelance copywriter, copywriter career path, how to become a copywriter in India, and copywriting career outlook. 

5 Jobs Available
Vlogger

In a career as a vlogger, one generally works for himself or herself. However, once an individual has gained viewership there are several brands and companies that approach them for paid collaboration. It is one of those fields where an individual can earn well while following his or her passion. 

Ever since internet costs got reduced the viewership for these types of content has increased on a large scale. Therefore, a career as a vlogger has a lot to offer. If you want to know more about the Vlogger eligibility, roles and responsibilities then continue reading the article. 

3 Jobs Available
Publisher

For publishing books, newspapers, magazines and digital material, editorial and commercial strategies are set by publishers. Individuals in publishing career paths make choices about the markets their businesses will reach and the type of content that their audience will be served. Individuals in book publisher careers collaborate with editorial staff, designers, authors, and freelance contributors who develop and manage the creation of content.

3 Jobs Available
Journalist

Careers in journalism are filled with excitement as well as responsibilities. One cannot afford to miss out on the details. As it is the small details that provide insights into a story. Depending on those insights a journalist goes about writing a news article. A journalism career can be stressful at times but if you are someone who is passionate about it then it is the right choice for you. If you want to know more about the media field and journalist career then continue reading this article.

3 Jobs Available
Editor

Individuals in the editor career path is an unsung hero of the news industry who polishes the language of the news stories provided by stringers, reporters, copywriters and content writers and also news agencies. Individuals who opt for a career as an editor make it more persuasive, concise and clear for readers. In this article, we will discuss the details of the editor's career path such as how to become an editor in India, editor salary in India and editor skills and qualities.

3 Jobs Available
Reporter

Individuals who opt for a career as a reporter may often be at work on national holidays and festivities. He or she pitches various story ideas and covers news stories in risky situations. Students can pursue a BMC (Bachelor of Mass Communication), B.M.M. (Bachelor of Mass Media), or MAJMC (MA in Journalism and Mass Communication) to become a reporter. While we sit at home reporters travel to locations to collect information that carries a news value.  

2 Jobs Available
Corporate Executive

Are you searching for a Corporate Executive job description? A Corporate Executive role comes with administrative duties. He or she provides support to the leadership of the organisation. A Corporate Executive fulfils the business purpose and ensures its financial stability. In this article, we are going to discuss how to become corporate executive.

2 Jobs Available
Multimedia Specialist

A multimedia specialist is a media professional who creates, audio, videos, graphic image files, computer animations for multimedia applications. He or she is responsible for planning, producing, and maintaining websites and applications. 

2 Jobs Available
Welding Engineer

Welding Engineer Job Description: A Welding Engineer work involves managing welding projects and supervising welding teams. He or she is responsible for reviewing welding procedures, processes and documentation. A career as Welding Engineer involves conducting failure analyses and causes on welding issues. 

5 Jobs Available
QA Manager
4 Jobs Available
Quality Controller

A quality controller plays a crucial role in an organisation. He or she is responsible for performing quality checks on manufactured products. He or she identifies the defects in a product and rejects the product. 

A quality controller records detailed information about products with defects and sends it to the supervisor or plant manager to take necessary actions to improve the production process.

3 Jobs Available
Production Manager
3 Jobs Available
Product Manager

A Product Manager is a professional responsible for product planning and marketing. He or she manages the product throughout the Product Life Cycle, gathering and prioritising the product. A product manager job description includes defining the product vision and working closely with team members of other departments to deliver winning products.  

3 Jobs Available
QA Lead

A QA Lead is in charge of the QA Team. The role of QA Lead comes with the responsibility of assessing services and products in order to determine that he or she meets the quality standards. He or she develops, implements and manages test plans. 

2 Jobs Available
Structural Engineer

A Structural Engineer designs buildings, bridges, and other related structures. He or she analyzes the structures and makes sure the structures are strong enough to be used by the people. A career as a Structural Engineer requires working in the construction process. It comes under the civil engineering discipline. A Structure Engineer creates structural models with the help of computer-aided design software. 

2 Jobs Available
Process Development Engineer

The Process Development Engineers design, implement, manufacture, mine, and other production systems using technical knowledge and expertise in the industry. They use computer modeling software to test technologies and machinery. An individual who is opting career as Process Development Engineer is responsible for developing cost-effective and efficient processes. They also monitor the production process and ensure it functions smoothly and efficiently.

2 Jobs Available
QA Manager
4 Jobs Available
AWS Solution Architect

An AWS Solution Architect is someone who specializes in developing and implementing cloud computing systems. He or she has a good understanding of the various aspects of cloud computing and can confidently deploy and manage their systems. He or she troubleshoots the issues and evaluates the risk from the third party. 

4 Jobs Available
Azure Administrator

An Azure Administrator is a professional responsible for implementing, monitoring, and maintaining Azure Solutions. He or she manages cloud infrastructure service instances and various cloud servers as well as sets up public and private cloud systems. 

4 Jobs Available
Computer Programmer

Careers in computer programming primarily refer to the systematic act of writing code and moreover include wider computer science areas. The word 'programmer' or 'coder' has entered into practice with the growing number of newly self-taught tech enthusiasts. Computer programming careers involve the use of designs created by software developers and engineers and transforming them into commands that can be implemented by computers. These commands result in regular usage of social media sites, word-processing applications and browsers.

3 Jobs Available
Product Manager

A Product Manager is a professional responsible for product planning and marketing. He or she manages the product throughout the Product Life Cycle, gathering and prioritising the product. A product manager job description includes defining the product vision and working closely with team members of other departments to deliver winning products.  

3 Jobs Available
Information Security Manager

Individuals in the information security manager career path involves in overseeing and controlling all aspects of computer security. The IT security manager job description includes planning and carrying out security measures to protect the business data and information from corruption, theft, unauthorised access, and deliberate attack 

3 Jobs Available
ITSM Manager
3 Jobs Available
Automation Test Engineer

An Automation Test Engineer job involves executing automated test scripts. He or she identifies the project’s problems and troubleshoots them. The role involves documenting the defect using management tools. He or she works with the application team in order to resolve any issues arising during the testing process. 

2 Jobs Available
Back to top